Download as pdf or txt
Download as pdf or txt
You are on page 1of 75

MCQs

Ear
Anatomy of the ear
1- The first visceral cleft gives all of the following except
a. Outer layer of the tympanic membrane
b. The auricle.
c. The external auditory canal.
d. The fibrous layer of the tympanic membrane.

2- The first visceral arch gives all of the following except


a- outer layer of the tympanic membrane
b- malleus.
c- incus .
d- the fibrous layer of the tympanic membrane.

3- The stapes bone develops from


a. First visceral arch.
b. Second visceral arch.
c. The otic capsule.
d. Both b & c are right.
e. Both a & b are right.

4- The inner ear is ebmbryologically derived from


a. Ectoderm
b. Endoderm.
c. Mesoderm.
d. all of the above .

5- A poorly developed anihelix and overdeveloped conchal cartilage


are characteristic of:
a. Darwin’s tubercle.
b. Microtia.
c. Bat ear.
d. Cauliflower ear.
e. Congenital syphilis.

6- The length of the bony external canal of adult person is approximately:


a. 8mm.
b. 12 mm.
c. 16 mm.
d. 24 mm.

7- The tympanic membrane takes its innervations from


a. V cranial nerve
b. X cranial nerve
c. XI cranial nerve
d. All of the above
e. Both a and b are true

8- The auriculo-temporal nerve


a. supplies the skin of the upper 2/3 of the lateral surface of the auricle
b. is a branch of maxillary nerve
c. supplies the middle ear mucosa through the tympanic plexus
d. none of the above.

9- The lobule of the ear has its sensory innervations from


a. Great auricular nerve
b. auriculo-temporal nerve
c. Aarnold branch of vagus nerve
d. facial nerve .

10- The tympanic membrane is divided into:


a. Two equal parts called pars tensaand pars flaccida
b. A major upper part called pars flaccida and a small lower part
called pars tensa
c. A small upper part called pars flaccida and a major lower part
called pars tensa
d. none of the above .
11-The bulge seen on the medial wall of the middle ear is
a. formed by the bony semi-circular canal.
b. known as the promontory.
c. is formed by the basal turn of the bony cochlea
d. both a and b are true
e. both b and c are true.

12- The oval window


a. Lies below and behind the promontory
b. is closed by secondary tympanic membrane
c. Leads to the scala tympani of the cochlea
d. is Closed by foot plate of stapes.

13- In the adult, the Eustachian tube is approximately the following length
a. 30mm
b. 20mm
c. 36mm
d. 45mm.

14- The Eustachian tube is opened by contraction of


a. tensor tympani muscle
b. levator palati muscle
c. tensor palati muscle
d. Salpingopharyngeus muscle.

15- The Eustachian tube


a. opens in the lateral wall of the oropharynx
b. is opened by the levator palati muscle
c. wider in the adult than in infants
d. non of the above
e. all of the above.

16- In the adult, the bony part of the Eustachian tube is approximately
the following length
a. 30 mm.
b. 12 mm.
c. 36 mm.
d. 45 mm.
17- The internal carotid artery comes in relation to which wall of the middle ear
a. Anterior wall
b. Roof
c. Inferior wall
d. Posterior wall.

18- The sensory end-organ of the semicircular canal is


a. the organ of Corti
b. the macula
c. the crista
d. non of the above

19- Which of the following structures does not pass through the internal auditory meatus
a. VII cranial nerve
b. VIII cranial nerve
c. Chorda tympani nerve.
d. Internal auditory artery.

20- The ratio of the functioning area of the tympanic membrane to the foot plate of stapes is
a. 15:1
b. 17:1
c. 23:1
d. 1:10

21- The sensory end-organ of the cochlea is


a. the organ of Corti
b. the macula
c. the crista
d. none of the above.

22- The content of the membranous labyrinth is


a. Endolymph
b. Perilymph
c. CSF
d. All.

23- Which of the following cranial nerves does not provide sensory
nerve supply to the auricle and external auditory canal?
a. V cranial nerve
b. VII cranial nerve
c. IX cranial nerve
d. X cranial nerve

24- McEwen's triangle is the surface landmark of:


a. The tympanic part of the facial nerve.
b. Mastoid antrum.
c. Dome of the lateral semicircular canal.
d. Incudo-stapedial joint.

25- The concept that the facial nerve supplies the auricle is related to:
a. Ramsy-Hunt syndrome.
b. Jugular foramen syndrome.
c. Horner's syndrome.
d. Bell's palsy.

26- The internal auditory canal


a. Connects the cerebell-pontine angle to the middle ear.
b. is 24 mm long
c. Contains the facial nerve and VIII nerve
d. Directed downwards, forwards and laterally .
EXAMINATION OF THE EAR

1- During ear examination the reservoir sign is diagnostic of


a. acute otitis media
b. mastoiditis
c. Petrositis.
d. All of the above.

2- Greisinger’s sign means:


a. Pain over the tempromandibular joint
b. Pain in the eye .
c. Pain and tenderness over the posterior part of the mastoid .
d. Pain and tenderness over the auricle.

3- Griessinger’s sign is positive in:


a. Acute petrositis .
b. Acute sinusitis .
c. Acute labyrinthitis .
d. Lateral sinus thromophelbitis .

4- Retracted drum is characterized by all the following except:


a. Disturbed cone of light .
b. Prominent malleolar folds
c. Decreased drum mobility .
d. Central drum perforation.

5- In clinical practice, the 512-Hz tuning fork has traditionally been preferred.
a. This frequency provides the best balance of time of tone
decay and tactile vibration.
b. Lower-frequency tuning forks like the 256-Hz tuning
fork provide greater tactile vibration.
c. Lower Frequency forks are better felt than heard.
d. All are true

6- In a normally hearing person


a. Rinne’s test is negative.
b. Air conduction is better than Bone conduction.
c. Air conduction is equal to bone conduction
d. None of the above.

7- Rinne’s test for the left ear is negative in


a. Left Conductive deafness.
b. left severe sensorineural deafness.
c. Bilateral conductive deafness.
d. All of the above .

8- Weber’s test compares


a. Air conduction in both ears.
b. Bone conduction of both ears.
c. Air conduction and bone conduction in one ear
d. Bone conduction of patient and the examiner.

9- Weber test is
a- Central in normal hearing
b- Lateralized to the side of conductive deafness
c- Lateralised to the normal side in cases of unilateral sensorineural heaing loss
d- All are true

10- In a patient with right conductive deafness, the sound of the


tuning fork placed on the forehead is
a. Lateralized to the left side.
b. Lateralized to the right side.
c. Sound is equally heard in both ears.
d. None of the above.

11- Rinne’s test is negative in all of the following EXCEPT


a. Stapedial Otosclerosis
b. Wax impaction.
c. Ossicular disruption
d. Meniere’s disease.

12- The objective method for diagnosis of conductive hearing loss is:
a. Pure tone audiometry.
b. Tympanometry.
c. Speech audiometry.
d. Auditory brain stem response.
e. Tuning fork tests.
13- The objective method for diagnosis of sensori-neural hearing loss is:
a. Pure tone audiometry.
b. Speech audiometry.
c. Tympanometry.
d. Auditory brain stem response.

14- Which of the following modalities of hearing assessment


does not require a response from the patient?
a. Whisper test
b. Rinne and Weber tuning fork test
c. Pure tone audiogram
d. Otoacoustic emission
e. Speech testing

15- The landmarks of the tympanic membrane on clinical examination


include all of the following except
a. The cone of light.
b. The tympanic annulus
c. The handle of malleus.
d. The foot plate of stapes.

16- Vertigo and nystagmus induced by pressure on the tragus is diagnostic of


a. serous labyrinthitis
b. circumscribed per-labyrinthitis
c. suppurative labyrinthitis
d. all of the above.

17- A positive fistula test is diagnostic of


a. serous labyrinthitis.
b. circumscribed peri-labyrinthitis
c. suppurative labyrinthitis.
d. Oro-antral fistula.

18- Type c tympanogram is consistent with


a. secretory otitis media
b. otosclerosis
c. Eustachian tube dysfunction
d. Ossicular discontinuity.

19- The expected type of tympanogram in secretory otitis media is


a. Type A
b. Type B
c. Type C
d. Type As
e. Type Ad

20- in 40 years female patient with left hearing loss, Rinne's test is
negative on the left side and tympanogram is type As, your diagnosis is
a. Left secretory otitis media.
b. Left otosclerosis.
c. Left tympanosclerosis
d. All are true.

21- On ear examination a red mass seen behind the tympanic membrane
which blanches on compression by pneumatic otoscope. This sign is called
a. Griesinger’s sign
b. Schwartz sign
c. Brown’s sign
d. Moor’s sign.

22- Fever in lateral sinus thrombosis is usually:


a. Intermittent.
b. Remittent.
c. Low grade.
d. High grade.

23- The discharge in case of cholesteatoma is:


a. Copious purulent.
b. Copious offensive.
c. Scanty offensive.
d. Thick scanty creamy.

24- which one is NOT true about pure tone audiometry :


a. It gives the amount of the hearing loss in dB. .
b. It gives the type of deafness.
c. It helps in hearing aid selection.
d. It helps in follow up of the case.
e. It measures the sound emitted from the cochlea.

25- which one is not true about impedance audiometry:


a. It measures the pressure changes in the middle ear.
b. It measures fixation and dislocation of the ossicular chain.
c. It measures the patency of the Eustachian tube.
d. It measures the sound emitted from the cochlea.
26- The test of hearing in infants is:
a. Weber's test.
b. Pure tone audiometry.
c. ABR "Auditory Brain stem Response".
d. All .

27- ABR "Auditory Brain stem Response" is used in:


a. Test of hearing in malingering.
b. Test of hearing in retro-choclear lesion.
c. Detection of acoustic neuroma.
d. All of the above.

28- Which is not true about CSF examination in case of meningitis:


a. Protein diminished.
b. Sugar diminished..
c. Cell count increased.
d. pressure increased.

29- Inspite of a perilabyrinthine fistula being present , the test is negative in cases of:
a. Labyrinthine fistula with dead ear.
b. Cholesteatoma bridging an inner ear fistula.
c. .Hyper mobile footplate of the stapes.
d. a and b .

30- Reservoir sign is positive in:


a. Acute petrositis
b. Chronic labyrinthitis
c. Acute mastoiditis
d. Cholesteatoma.

31- By central drum perforation we mean :


a. perforation at the central part of the drum
b. a perforation in the pars tensa which is surrounded by a rim of tympanic membrane.
c. a perforation of the pars flaccida
d. a perforation in the pars tensa which is not surrounded by a rim of tympanic membrane.

32- The nystagmus in right caloric test when using warm water has
a. a slow component to the left side
b. a slow component to the same side
c. vertical direction
d. pendular

33- The tuning fork tests


a. are used to assess the degree of hearing loss.
b. Help to differentiate between conductive and sensori-neural hearing loss.
c. Both a and b are true
d. are not used nowadays and replaced by audiometry.

34 - false +ve fistula test is due to


a. abyrinthine fistula with dead ear.
b. holesteatoma bridging an inner ear fistula.
c. .Hyper mobile footplate of the stapes.
d. ll of the above.
The PAINFUL EAR

1. Pain is a symptom of
a- Acute otitis media
b- Chronic otitis media
c- Impacted wax
d- Otitis media with effusion

2. The causative organism in ear furuncle is


a. proteus
b.Pseudomonas
c. Staph. Aeureus
d.E coli.

3. Malignant otitis externa is


a- a malignant tumor affecting the external auditory canal in
elderly uncontrolled diabetic patients
b- is a potentially fatal disease
c- it is also called skull base osteomyelitis.
d- all are true.

4. Itching of the ear is a characteristic symptom of:


a. Cholesteatoma
b. Acute suppurative otitis media
c. Otosclerosis
d. Otomycosis

5. The following organisms are involved in acute otitis media except


a. Streptococcus pneumonia
b. Hemophilus influenza
c. Pseudomonas aeroginosa
d. Morexella catarrhalis.

6. Throbbing and severe earache is present in the following stage of acute otitis media
a. stage of salpingitis
b. stage of catarrhal otitis media
c. stage of suppurative otitis media
d. stage of tympanic membrane perforation.

7. Most cases of extradural abscess of the temporal lobe


a. are asymptomatic and discovered accidentally during mastoidectomy.
b. present with persistent ipsilateral temporal headache.
c. present with vertigo.
d. present with pulsating discharge, hearing loss and tinnitus.

8. In a patient having acute suppurative otitis media with bulging drum,


myringotomy is beneficial to
a. drain the middle ear
b. avoid rupture of the tympanic membrane
c. avoid complications
d. all of the above .

9. Uncontrolled diabetes in an elderly patient may predispose to


a. cholesteatoma
b. malignant otitis externa
c. presbyacusis
d. vestibular neuronitis.
10. In malignant otitis externa all the following is true EXCEPT:
a. It is common in old diabetic.
b. There may be facial paralysis.
c. The commonest organism is pseudomonas.
d. Mainly treated surgically.

11. The management of malignant otitis externa depends on


a) Strict control of blood sugar in diabetic patient
b) Antibiotics in large dose
c) Local debridement of any necrotic tissue
d) All

12. A patient with long standing left otorrhoea presented with


persistent left temporal headache, the diagnosis may be
a. migraine.
b. mastoiditis
c. extradural abscess.
d. meningitis.

13. In ottic barotrauma, the following statements are correct EXCEPT


a. occurs during airplane ascent
b. occurs during airplane rapid descent
c. can cause rupture of the tympanic membrane
d. occurs during diving.

14. Auricular hematoma


a. may be complicated by otitis externa
b. cauliflower ear is one of its complications.
c. evacuation of the extra-vasated blood is not essential
d. all of the above.

15. A slippery foreign body in the external canal must not be


removed by forceps because
a. It may slipper deeper in the external canal
b. It may injure the tympanic membrane during its removal
c. It is usually difficult to grasp and remove by this method
d. All of the above.

16. An impacted seed in the external canal is best removed by


a. Ear wash
b. Suction
c. Hook or crocodile forceps
d. None of the above.

17. Longitudinal temporal bone fracture :


a. is less common than the transverse type
b. is usually associated with sensori-neural hearing loss
c. facial nerve paralysis is a common association with this type
d. none of the above.

18. Conductive deafness in longitudinal temporal bone fracture may be due to:
a. Rupture of the tympanic membrane
b. Ossicular disruption.
c. None of the above.
d. Both a and b

19. In traumatic rupture of the drum, which of the following is true:


a. It usually heals spontaneously within 3 months.
b. Local ear drops are highly indicated
c. Myringoplasty is the first line of treatment.
d. Suction of blood clots help healing.

20. The pain in acute suppurative otitis media in the suppurative stage is:
a. Dull aching.
b. Throbbing.
c. Boring.
d. Burning.

21. Pain in acute tonsillitis is referred to the ear through:


a. The 5th nerve.
b. The 9th nerve.
c. The 10th nerve.
d. 2 nd & 3rd cervical nerve.
22. Pain in acute sinusitis is referred to the ear through:
a. The 5th nerve.
b. The 9th nerve.
c. The 10th nerve.
d. 2 nd & 3rd cervical nerve.

23. Pain in tempro-mandibular joint is referred to the ear through:


a. The 5th nerve.
b. The 9th nerve.
c. The 10th nerve.
d. 2 nd & 3rd cervical nerve.

24. Pain in cancer larynx is referred to the ear through:


a. The 5th nerve.
b. The 9th nerve.
c. The 10th nerve.
d. 2 nd & 3rd cervical nerve.

25. Pain in salivary calculi is referred to the ear through:


a. The 5th nerve.
b. The 9th nerve.
c. The 10th nerve.
d. 2 nd & 3rd cervical nerve.

26. The pain in acute suppurative otitis media is more severe at:
a. Night.
b. Morning.
c. Mid-day.
d. All the day.

27. Which is a risk factor for otitis media in infants.


a. Bottle feeding
b. Premature birth
c. Being first born
d. Diarrhea.

28. Unilateral referred otalgia is due to all of the following except:


a. Peritonsillar abscess.
b. Ulcer oral tongue.
c. Allergic rhinitis.
d. Tempromandibular joint dysfunction.
e. Cancer of pyriform fossa.

29. Referred otalgia after tonsillitis is along


a. Jackobson nerve
b. Arnold nerve
c. Facial nerve
d. Auriculotemporal nerve

30. Haematoma auris may be complicated by


a. Bleeding from the ear
b. Perichondritis
c. Otitis externa
d. All

31. hematoma auris


a. is due to extravasation of blood in the subcutaneous tissue
b. is treated by warming the pinna
c. It can lead to loss of cartilage support of the pinna
d. Hemostasis, bandage and follow up are suffucent .

32. A 50 year old male patient presented with otalgia, on examination


both external auditory canal and tympanic membrane are normal,
all of the following might be the sites of origin of his pain EXCEPT
a. Teeth
b. Tongue
c. Cervical vertebrae
d. Inner ear .

33. The earliest manifestation of glomus tympanicum is


a. conductive deafness
b. pulsating tinnitus
c. bleeding from the ear
d. bleeding polyp in the external auditory canal.
e. Ear pain.

34. Pain is a characteristic symptom of


a. bullous myringitis
b. chronic suppurative otitis media
c. otitis media with effusion
d. otosclerosis
35. Which is TRUE about malignant otitis externa
a. painless otorrhea is charecterstic feature
b. also called skull base osteomyelitis
c. caused by streptococci
d. the first line of treatment is surgical

36. A 4-year-old boy is taken to the hospital, with a history of left ear
ache and Oral vomiting for 1 day. On inspection His pinna is pushed
downwards and forwards and a small perforation leaking pus is
noted in the pars tensa. His temperature is 39.2C. 1.
What is the next best step in the management of this patient?
a. Oral Amoxicillin
b. Emergency Mastoidectomy
c. Amoxil and clavulanic acid
d. Admission and IV antibiotic

37. A 70-year-old woman has weakness of the left side of her face.
She has had a painful ear for 48 hours. There are vesicles in the
left ear canal and on the ear drum. 24. Which of the following is
associated with this condition?
a- Paracusis willisii
b- Furunculosis of the external auditory meatus
c- osteomyelitis of the temporal bone caused by pseudomonas
d- Paraganglioma arising from the middle ear
e- Reactivation of a Latent viral infection in the geniculate ganglion

38. Which is not true regarding malignant otitis externa


a. It is spreading osteomyelitis caused by hemophylus influenza
b. A purulent discharge comes through the tympanic membrane
c. Night ear pain is a characteristic symptom
d.Prognosis is usually good.

39. A two years old boy is suffering from acute otitis media.
His pain is not relieved inspite of adequate treatment. The best treatment is
a. Change of antibiotic
b.Increase analgesic dose
c. Give analgesic ear drops
d.Refer for myringotomy.

40. Which of the following is not a feature of otitic barotrauma


a-Otalgia
b-deafness
c-Tinnitus
d-Purulent ear discharge.

41. A 30 y old lady c/o of severe otalgia and small vesicles on the
concha. One day later she suffered dropping of the
angle of the mouth on the same side. The diagnosis is
a. Ramsay Hunt syndrome
b.Bullous myringitis
c. Glomus tumour
d.Acute otitis media.

42. In management of auricular hematoma


a. Aspiration of blood is only needed
b. Systemic antibiotic is sufficient
c. Evacuation of blood followed by pressure bandage is the proper management
d. No treatment is needed because it will resolve spontaneously.

43. A 64 years old diabetic patient with severe otalgia that is


unrelieved by many analgesics. examination of the ear shows
granulation tissue in the external auditory canal.
a. Chronic suppurative otitis media
b.Fungal otitis externa
c. Ramsy Hunt syndrome
d.Malignant otitis externa
e. Furunculosis.

44. A 45 years old presents with severe dull pain in the right ear that is
radiating to the pinna. The patient reported ringing in the ear . on
examination there are small blisters over the auricle and in the external
auditory canal.
a. Chronic suppurative otitis media
b.Fungal otitis externa
c. Ramsy Hunt syndrome
d.Malignant otitis externa
e. Furunculosis
45. Regarding otomycosis
a. Systemic treatment is necessary
b. Aspirigillus and candida are the most common responsible organisms
c. Otalgia is the main complaint
d. Occurs commonly in diabetics

46. In a senile patient with uncontrolled diabetes, examination


of the external canal reveled granulation tissue in the
floor of the external auditory canal. If CT scan and MRI are NEGATIVE ,
you should proceed to
a. CT can
b. MRI
c. Bone scan
d. Biopsy.

47. A 38-year-old man with diabetes mellitus type 2 has just returned
from holiday where he went swimming every day. For the last few days
he has had irritation in both ears. He has no facial paresis, but his right ear
is hot, red, and acutely painful. The external meatus appears swollen, but his
tympanic membrane is mobile. What is the most likely diagnosis?
a. Malignant otitis externa
b. Furuncle in the external meatus
c. Diffuse Otitis Externa
d. Acute otitis Media
e. Ramsy-Hunt syndrome.

48.Rupture of the tympanic membrane issuspected if during ear wash there is


a- Sudden Pain
b- Sensation of water in the throat
c- Blood tinged returning water
d- All.

49. A 65-year-old woman presents with severe otalgia that is unrelieved


by oral analgesics. On inspection the patient has a facial paresis.
Examination of the left ear is painful and shows granulation tissue in
the external auditory canal but minimal debris.
A- What is the MOST likely diagnosis?
a. Malignant Otitis Media
b. Diffuse Otitis externa
c. Necrotizing Otitis Externa
d. Acute suppurative otitis Media
e. Chronic suppurative Otitis media

B- Management typically involves all the following EXCEPT


a. Monitoring the patients ESR
b. Glucose control
c. Corticosteroids
d. IV fluoroquinolone therapy
e. Aural toilet

50. A 45-year-old man presents to the clinic with dull aching pain and
tinnitus to the right ear. On examination there are small blisters
over the auricle and within the external auditory canal. He has
difficulty closing the right eye and keeping food in his mouth while chewing.
Treatment of choice in his condition is
a. Mastoidectomy
b.Botulinum toxin injections
c. acyclovir & steroids
d.antibiotics directed against Pseudomonas
e.surgical resection of the geniculate ganglion.

51. The symptom always present in bullous myringitis is


a- Facial nerve paralysis
b- Fever
c- Ear pain
d- Purulent ear discharge
Discharging ear

1- All of the following may be seen in the tubotympanic type of chronic


suppurative otitis media except
a. mucopurulent otorrhoea
b. central tympanic membrane perforation
c. marginal tympanic membrane perforation
d. profuse otorrhoea .

2- Cholesteatoma is characterized by
a. continuous mucopurulent ear discharge
b. A foul smelling ear discharge
c. A central tympanic membrane perforation
d. none of the above.

3- In a patient suffering from purulent otorrhoea and attic perforation


a. treatment is essentially surgical
b. medical treatment and follow up is sufficient
c. myringoplasty is the only needed treatment
d. none of the above.

4- The discharge in case of cholesteatoma is:


a. Copious purulent.
b. Copious offensive.
c. Scanty offensive.
d. Thick scanty creamy.

5- A patient with uncomplicated CSOM has:


a. Ear discharge & headache.
b. Ear discharge & dizziness
c. Ear discharge & hearing impairment.
d. Ear discharge & fever.

6- Pulsating ear discharge may be found in:


a. Extradural abscess.
b. Acute exacerbation of CSOM.
c. Acute otitis media with small perforation.
d. All of the above.

7- All the following are causes of pulsating otorrhoea except


a. Acute suppurative otitis media
b. Cholesteatoma
c. Acute exacerbation of chronic suppurative otitis media
d. Chronic otitis media complicated by extradural abscess.

8- In furunculosis
a. is caused by streptococci
b. Tenderness over the mastoid is the predominant complaint.
c. Ear discharge if there is scanty, cheesy purulent
d. Incision and drainage is usually required.

9- Acute mastoididis
a. is most common in young children
b. The commonest presenting sign is mastoid swelling with anterior
displacement of the pinna
c. Cortical mastoidectomy is essential if mastoid abscess has formed
d. Reservoir sign is characteristic
e all

10- A patient with a history of right discharging ear for more than 10
years presented with right deep seated earache, offensive otorrhoea
and fleshy mass in the external canal. You must suspect
a. glomus tympanicum tumor.
b. Malignant otitis externa.
c. Squamous cell carcinoma.
d. Tubotympanic chronic suppurative otitis media with an aural polyp.

11- The characteristic feature of cholesteatoma is


a. Continuous mucopurulent discharge
b. Foul smelling ear discharge
c. An intermittent ear discharge
d. Hearing.

12- Systemic antibiotic is preferred in treatment of


a. Acute suppurative otitis media
b. Chronic suppurative otitis media
c. Cholesteatoma
d. Otitis media with effusion .

13- A 15 years old with right offensive otorrhea, temperature 38 C,


left body weakness, visual field defect is suffering from
a- Left cerebellar abscess
b- Right cerebellar abscess
c- Right temporal temporoparietal lobe abscess
d- Left temporal temporoparietal lobe abscess.
14- Offensive otorrhea with marginal drum perforation is managed by
a. Local ear drops
b. Systemic antibiotics & local drops
c. Regular follow up
d. Tympanomastoid exploration

15- Reservoir sign is positive in :


a. Acute periostis
b. chronic labryinthitis
c. acute mastoidits
d. cholesteatoma
e. non of the above

16-In a patient suffering from purulent otorrhoea and attic perforation


a. treatment is essentially surgical
b. medical treatment and follow up is sufficient
c. myringoplasty is the only needed treatment
d. none of the above.

17- Uncontrolled diabetes in elderly patient may predispose to


a. cholesteatoma
b. malignant otitis externa
c. presbyacusis
d. vestibular neuronitis.

18- A child with retraced drum and conductive deafness after


inadequate treatment of acute suppurative otitis media is suffering from
a. chronic tubotympanic otitis media
b. chronic atticoantral otitis media
c. otitis media with effusion
d. all of the above
e. none of the above .

19- The tympanic membrane perforation in acute otitis media is


a. central in the pars tensa
b. marginal in the pars tensa
c. small in the pars flaccida
d none of the above.

20- Before tympanoplasty in a 30 years old patient, the following is required


a. Audiogram
b. ensure dry perforation
c. treatment of any underlying nasal or paranasal sinus infection
d. all of the above
e. none of the above.

21- The aim of radical mastoidectomy is:


a. To give safe ear.
b. To preserve hearing.
c. Reconstruct the ossicles.
d. Reconstruct the tympanic membrane.

22- By radical mastoidectomy operation we mean


a. removal of mastoid air cells and all middle ear contents except stapes footplate.
b. removal of diseased mastoid air cells
c. removal of mastoid air cells and all middle ear contents with
preservation of healthy remnants of tympanic membrane and ossicles.
d. none of the above.

23- Which of the following statement is true about necrotizing otitis externa
a. Staph is the commonest causative organism
b. It is seen in any age
c. Potentially fatal
d. Surgical intervention is never needed

24- Treatment of cholesteatoma of the middle ear in a 5 years child is


a. Modified radical mastoidectomy
b. Medical treatment and follow up
c. cortical mastoidectomy
d. none.

25- CSF otorrhea


a. may be secondary to a temporal bone fracture in conjunction
with a perforated TM
b. With an intact TM, a CSF leak may manifest as rhinorrhea if the CSF
travels down the eustachian tube.
c. Traumatic CSF otorrhea is often managed conservatively
d. β2-transferrin if enough fluid can be collected is diagnostic .
e. All

26- Pulsatile otorrhea seen in


a. Glomus tumour
b. CSF otorrhea
c. ASOM
d. fistula

27- If pain and discharge persists after acute otitis media even after
appropriate antibiotic, one should suspect
a- Mastoiditis
b- Extradural abscess
c- Meningitis
d- Petrositis

28- CSF otorrhea after temporal bone fracture is managed by


a. antibiotics that cross the blood brain barrier
b. coverage of the ear by sterile gauze
c. avoid straining
d. all of the above
e. non of the above .
29- A 40-year-old man presents with a history of unilateral offensive ear
discharge for 6 months with squamous debris in the epitympanum
that is not self-cleaning. A perforation is seen in the pars flaccida.
What is the most likely diagnosis?
a. Squamous cell carcinoma
b. Chronic otitis media Atticoantral type
c. Chronic otitis media tubotympanic type
d. Foreign body (e.g. cotton bud) in the ear
e. Malignant otitis externa

30-In surgical treatment of cholesteatoma , the first priority is


a- Making ear dry
b- Improve hearing
c- Preservation of hearing
d- Rendering the ear safe

31- A 40-year-old man presents with a history of unilateral offensive ear


discharge for 6 months with squamous debris in the epitympanum
that is not self-cleaning. A perforation is seen in the pars flaccida.
The next best step in managing this patient is
a. Radiotherapy
b. Topical antimicrobial therapy
c. Tympanomastoid surgery
d. Audiometry & Temporal Bone CT
e. Syringing of the ear canal.

32- Acute necrotizing otitis media is a complication of


a- Measles
b- Mumps
c- Diabetes
d- Pseudomonal infection

33- Cholesteatoma commonly causes a fistula in the


a- Promontory
b- Lateral semicircular canal
c- Stapes foot plate
d- Facial nerve canal

34- A large near total perforation following acute necrotizing otitis


media must be followed up for fear of
a- recurrent middle ear infection.
b- secondary acquired cholesteatoma .
c- retraction pocket.
d- Tympanosclerosis.
The deaf ear

1) A child after inadequate treatment of acute suppurative otitis


media is still suffering impaied hearings and foreshortened handle of malleus.
a. Tympanometry is needed
b. He is most probably suffering otitis media with effusion.
c. This may be a case of unresolved acute otitis media
d. all of the above.

2) A 5-year-old child has persistent serous effusions in both ears for 6


months after a routine acute infection. He has a 40-dB conductive
hearing loss in both ears and having trouble in school.
What would be the BEST treatment for this child?
a. observe the child for another 3 months
b. prescribe amoxicillin for 10 days
c. recommend hearing aids
d. place ventilating tubes
e. prescribe prophylactic antibiotics for 3 months.

3) All of the following are diagnostic of tympanic membrane retraction except


a. fore-shortened handle of malleus .
b. prominent lateral process of malleus.
c. Schwartz sign .
d. distorted cone of light.

4) Otitis media with effusion


a. is most prevalent in 2nd decade of life.
b. more likely in the presence of large infected adenoids.
c. Pain is a common feature.
d. Vertigo is a prominent feature .

5) The earliest manifestation of glomus tympanicum is


a. conductive deafness
b. pulsating tinnitus
c. bleeding from the ear
d. bleeding polyp in the external auditory canal.
e. all of the above.

6) In a 45 years old female patient presenting with pulsating tinnitus


and red mass behind the drum, all of the following are true EXCEPT
a. Glomus tumor is a possible diagnosis.
b. more assessment is needed by CT scan or MRI.
c. MRI angiography confirm the diagnosis .
d. biopsy is essential to verify the pathological nature.

7) The best investigation for diagnosis of vestibular schwannoma is


a. CT scan.
b. MRI
c. ABR
d. Pure tone audiogram

8) In a 50 years old patient presenting with right hearing loss and tinnitus of one
month duration. His pure tone audiogram showed right sensorineural hearing loss.
a. Presbyacusis is suspected
b. Nothing is done and annual follow up is only needed.
c. MRI is the best investigation to uncover the diagnosis.
d. All
9) It is better to AVOID ear wash in
a. Perforated ear drum
b. disc battery.
c. impacted vegetable foreign body
d. all

10) Which of the following statements is FALSE concerning myringotomy Operation


a. it is indicated in acute suppurative otitis media with bulging drum
b. it is indicated in secretory otitis media after failure of medical treatment
c. it is better done in the postero superior quadrant of the tympanic membrane.
d. Residual perforation of the tympanic membrane is one of its complications.

11) The most common complication of myringotomy operation is


a. injury of facial nerve.
b. dislocation of the incus
c. injury of the jagular bulb
d. residual perforation

12)In Myringotomy operation, the postero-superior quadrant of the


tympanic membrane must be avoided.
a. to avoid injury of dehiscent jagular bulb.
b. to avoid injury of the ossicles .
c. non of the above
d. both 1& 2.

13) The first line of treatment in a child who develops lower motor neurone facial
paralysis after acute otitis media is
a. antibiotics and corticosteroids.
b. decompression of facial nerve.
c. exploration of facial nerve
d. myringotomy.

14) Cochlear implant is indicated in patients having


a. Bilateral conductive hearing loss
b. Bilateral sensori-neural hearing loss
c. Bilateral total hearing loss who can not gain benefit from hearing aids.
d. all of the above.

15) Myringotomy may be indicated in:


a. Secretory otitis media
b. Acute otitis media with bulging drum
c. Otitic barotrauma
d. All of the above .

16) Most cases of sudden sensorineural hearing loss are


a. temporal bone fracture.
b. Genetic
c. Neurologic
d. Idiopathic

17) The type of deafness in otosclerosis may be


a. conductive
b. sensorineural
c. mixed
d. all

18) Which of the following is associated with objective tinnitus


a. Meniere’s disease.
b. Ear wax impaction
c. Vestibular schwannoma
d. Palatal myoclonus
e. Otitis media with effusion
19) The commonest cause of unilateral sensori-neural hearing loss in children is
a. Ototoxic poisoning.
b. Measles
c. Mumps.
d. Rh. incompatibility
e. Syphilis.

20) Fluctuant SNHL usually occurs in:


a. Presbyacusis.
b. Meniere's disease.
c. Otosclerosis.
d. All of the above.

21)In a patient suffering from sudden severe vertigo lasting for 2 days
which resolves spontaneously, but hearing is normal, the diagnosis is likely
a. Meniere’s disease
b. Vestibular neuronitis
c. Labyrinthitis
d. Vestibular schwannoma.

22) The commonest cause of vertigo is


a. Meniere’s disease
b. Labyrinthitis
c. benign paroxysmal positional vertigo
d. ototoxicity.

23) The commonest cause of conductive deafness in adults is:


a. wax
b. secretory otitis media
c. otomycosis
d. otosclerosis.

24) The commonest cause of conductive deafness in children is:


a. wax
b. otitis media with effusion
c. otomycosis
d. otosclerosis.

25) Mixed hearing loss may be caused by one of the following:


a. Otosclerosis.
b. Meniere's disease.
c. Ear wax.
d. Acoustic neuroma.

26) Etiology for pulsatile tinnitus includes the following EXCEPT


a. Arterio-venous malformation of neck.
b. Otosclerosis.
c. Glomus jugulare tumors.
d. Hyperthyroidism.

27) Slowly progressive conductive deafness in middle aged


female with normal drum & Eustachian tube function is most probably due to:
a. Otitis media with effusion.
b. Otosclerosis.
c. Malingering.
d. Tympanosclerosis.

28)In Paracusis Weleshii


a. The patient hears better in quiet places.
b. The patient hears better in noisy places.
c. The patient has sensori-neural hearing loss.
d. The patient can not tolerate loud sound.

29) The IX, X and XI cranial nerves may be involved in all of the following except
a. acoustic neuroma
b. transverse temporal bone fracture.
c. malignant otitis externa
d. squamous cell carcinoma of the middle ear.
30) The following have an ototoxic effect except
a. frusemide
b. Streptomycin
c. amoxicilline
d. quinine.

31) All are correct about Meniere's disease EXCEPT


a. It is an endolymphatic hydrops.
b. Males are more affected than females.
c. The type of deafness is conductive.
d. Decompression of the labyrinth is indicated if the vertiginous attack is crippling.

32) Referred otalgia from pyriform sinus cancer is through:


a. IX cranial nerve
b. X cranial nerve
c. XII cranial nerve
d. VII cranial nerve .

33) Which of the following drugs are known to cause tinnitus?


a. Salicylates.
b. Loop Diuretics.
c. Aminoglycosides.
d. NSAID.
e. All of the above.

34) A 36 years old man with head injury in traffic road accident.
There is unilateral hearing loss. Audiogram shows conductive
hearing loss of 60 dB in all frequencies and type Ad tymapanogram..
a. Hemotympanum
b. tympanic membrane perforation.
c. ossicular dislocation
d. perilymph fistula

35) A 65 years old patient presents with mild tinnitus in both ears which is more
noticeable in night. There is long standing hearing impairment. A pure tone audiogram
shows bilateral sensorineural hearing loss that is worse at higher frequencies.
a. Noise –induced hearing loss
b. Presbyacusis
c. Ramsay Hunt syndrome
d. Vestibular schwannoma.

36) A 55 years old woman presents with a unilateral conductive


hearing loss and pulsatile tinnitus in the right ear.
Otoscopy reveals reddish blue appearance of the drum.
a. Noise –induced hearing loss
b. Cholesteatoma
c. Presbyacusis
d. Ramsay Hunt syndrome
e. Glomus tumor

37) Rubella causes the greatest cochlear damage when the maternal infection occurs
a. During the 1st trimester
b. During the 2nd trimester
c. During the 3rd trimester
d. Any time during pregnancy

38) Which of the following drugs IS NOT OTOTOXIC


a. Frusemide
b. Vancomycin
c. Cisplatin
d. Amoxicilline .

39) The diagnosis of an acoustic neuroma is often triggered by a patient’s symptoms.


The most common presenting feature of acoustic neuromas, occurring in
90 percent of patients, is
a. deafness
b. vertigo
c. facial weakness
d. tinnitus.
40)In sudden sensorineural hearing loss
a. appropriate referral and treatment decisions in the management of
SSNHL, recognizing it is a medical emergency requiring urgent care.
b. Corticosteroids are given after full investigations to establish the diagnosis
c. Betahistine is the treatment of choice
d. Antibiotic , reassurance are the only needed treatment.

41) A mother brings her 5-day old baby to have a hearing assessment.
She is concerned because She has a history of sensorineural hearing
loss and did not receive all her vaccinations. The most appropriate
test to order is
a. MRI brain
b. Brainstem evoked response
c. Tuning fork test
d. Pure tune audiometry
e. Behavioural audiometry.

42) A 35-year old woman has noticed difficulty hearing on


the telephone since delivering her 3 rd child. Otoscopy shows a pink
discoloration behind the left tympanic membrane. Which of the
following is most likely to be consistent with her condition?
a. Noise induced deafness
b. Facial nerve palsy
c. Type Ad curve on tympanogram
d. Improvement in hearing with background noise

43) A 10-year old is brought to the Otolaryngology clinic by his mother.


She complains that he started to watch the television at high
volumes and at times he doesn’t respond to her. His teacher
recommends that he gets his ears tested due to poor school
performance. The most likely finding on otoscopy is
a. Foreign body
b. An air fluid levels
c. Red inflamed tympanic membrane
d. Wax impaction
e. Flamingo pink tympanic membrane

44) A 30 y old lady with family history of hearing loss developed


hearing loss and tinnitus during pregnancy. She described better
heaing in noisy environment. Pure tone audiogram
showed mixed hearing loss. What is the most likely diagnosis
a- Stapedial Otoscleosis
b- Cochlear otosclerosis
c- Cochlear and stapedial otosclerosis
d- Acoustic neuroma

45) Which of the following conditions predispose to otitis media with


effusion
a- Cleft palate
b- Nasopharyngeal carcinoma
c- Adenoids
d- All

46) Brown’s sign is seen in


a- Glomus tumour
b- Otosclerosis
c- Acoustic neuroma
d- Meniere’s disease

47) The commonest cause of sensorineural hearing loss in old age is


a) Acoustic neuroma
b) Wax
c) Otitis media with effusion
d) Presbyacusis

48) Conductive hearing loss in a child with flat curve in tympanometry


is diagnostic of
a) Chronic tubotympanic otitis media
b) Cholesteatoma
c) Unresolved acute otitis media
d) Otosclerosis

49) Hyperacusis is seen in


a- Bell’s palsy
b- Meniere’s disease
c- Cochlear otosclerosis
d- Acoustic neuroma

50) Conductive hearing loss in a 40 years female with restricted


curve in tympanometry is diagnostic of
a) Chronic tubotympanic otitis media
b) Cholesteatoma
c) Unresolved acute otitis media
d) Otosclerosis
The traumatized ear
1- In ottic barotrauma, the following statements are correct EXCEPT
a. occurs during airplane ascent
b. occurs during airplane rapid descent
c. can cause rupture of the tympanic membrane
d. occurs during diving.

2- Diving may lead to otitic barotrauma during


a- Descent
b- Ascent
c- Immediately after diving is over
d- Two hours after diving is over.

3- In traumatic perforation of the tympanic membrane


a. blast rupture affects the pars flaccida
b. blood clot should be syringed out immediately.
c. Myringoplasty should be performed as early as possible.
d. Severe pain continues for several days
e. Conservative treatment is sufficient

4- In skull base fracture involving temporal bone


a. Longitudinal fractures are more common than transverse fractures
b. Transverse fractures commonly cause facial nerve paralysis
c. Ecchymosis over the mastoid area may be present
d. Sensorineural hearing loss is usually irreversible
e. All are true

5- A slippery foreign body in the external canal must not be removed


by forceps because
a. It may slipper deeper in the external canal
b. It may injure the tympanic membrane during its removal
c. It is usually difficult to grasp and remove by this method
d. All of the above.

6- An impacted seed in the external canal is best removed by


a. Ear wash
b. Suction
c. Hook
d. None of the above.

7- Longitudinal temporal bone fracture


a. is less common than the transverse type
b. is usually associated with sensori-neural hearing loss
c. facial nerve paralysis is a common association with this type
d. none of the above.

8- Conductive deafness in longitudinal temporal bone fracture may be due to:


a. Rupture of the tympanic membrane
b. Ossicular disruption.
c. None of the above.
d. Both 1 and 2

9- In traumatic rupture of the drum, which of the following is true:


a. The main treatment is conservative.
b. It usually heals spontaneously within 3 months.
c. Local ear drops are highly indicated
d. Myringoplasty is the first line of treatment.
e. a and b are true .

10- Ear wash is contraindicated in all of the following EXCEPT


a- Impacted wax
b- Impacted vegetable foreign body
c- Traumatic drum perforation.
d- Disc battery in the external auditory canal.

11- In a small traumatic tympanic membrane perforation,


the first line of treatment is
a- Suction of blood in the external canal.
b- Arrange the patient for immediate myringoplasty
c- Systemic antibiotics and local ear drops .
d- Follow up for several month is preferred

12-In transverse fractures of the petrous temporal bone, which


cranial nerve is most likely injured?
a. Trigeminal nerve
b. Abducent nerve
c. Facial nerve
d. Glossopharyngeal nerve

13- You may suspect tympanic membrane perforation during ear wash if
a. Patient feels sudden ear pain.
b. patient feels water in the throat
c. trickling of blood from the external canal is seen.
d. all.

14- An elliptical perforation in otherwise healthy tympanic membrane occurs in


a- Tubotympanic chronic otitis media
b- Cholesteatoma
c- Traumatic tympanic membrane perforation
d- Bullous myringitis

15- A 24 year old male comes in ENT opd with complain of pain
in ear and reduced hearing sensitivity for 1 day .
On taking proper history he told his father slapped him 1 day back , on
examination of ear we found Perforation in Eardrum. What is next step?
a. Tympanoplasty
b. Wait and watch
c. Symptomatic treatment
d. both B& C
Vertigo

1- Vertigo is
a. A condition where a person has the sensation of movement or of
surrounding objects moving when they are not.
b. feeling of being lightheaded..
c. feeling of being unbalanced
d. feeling of blackout.

2- The commonest cause of vertigo is


a. Meniere’s disease
b. Labyrinthitis.
c. benign paroxysmal positional vertigo.
d. Vestibular neuronitis
e. Hypotension

3- In benign paroxysmal positional vertigo (BPPV), the afflicted


semicircular canal in 85% to 95% of cases is the
a. Lateral SCC
b. Posterior SCC
c. Superior SCC
d. All

4- In vestibular neuritis
a. It is the most common cause of vertigo
b. A bacterial inflammatory etiology is thought to underlie vestibular neuritis.
c. Vestibular and cochlear components of the 8th cranial nerve are both affected
d. The sudden onset of severe symptoms can raise concerns of a central etiology.

5- Which of the following is characterized by vertigo without hearing loss:


a. Benign paroxysmal positional vertigo.
b. Vestibular neuronitis
c. Meniere’s disease
d. All of the above
e. a and b are true
f. b and c are true.

6- Meniere's disease:
a. is commonly bilateral.
b. is the commonest cause of vertigo.
c. The type of deafness is conductive.
d. A decompression of the labyrinth is indicated if the vertiginous attack is crippling.

7- A 40 years old man presented with severe attack of vertigo. He had a history
of upper respiratory tract infection a week before. No complain of hearing loss,
tinnitus or facial paralysis, the likely diagnosis is
a. Benig paroxysmal positional vertigo
b. Vestibular neuronitis
c. Meniere’s disease
d. Serous labyrinthitis.

8- Which of the following medications is recommended in the first trial


treatment of Ménière disease?
a. Betahistine
b. Metoprolol
c. Topiramate
d. Carbamazepine

9- Regarding Eply maneuver, one is NOT TRUE


a. It is used to treat BPPV
b. It is effective in treatment of vestibular neuronitis
c. works by allowing, displaced otoconia from the affected semicircular canal to be
relocated by using gravity, back into the utricle,
d. It is a safe and effective treatment.

10- Regarding vestibular neuronitis,


a. Antiemetics and vestibular suppressants are useful
b. The prolonged use of vestibular suppressants may impede the
process of central vestibular compensation.
c. Most patients recover well, even without treatment.
d. any patient thought to have vestibular neuritis who has significant vascular
risk factors should be evaluated for possible stroke.
e. All are true.

11- Fistula test


a. Is a bedside vestibular method of examination that detects the
existence of an abnormal communication between the middle and inner ear.
b. Nystagmus slow phase is directed to the other side
c. Direct the attention to cholesteatma
d. is negative in dead labyrinth
e. All are true .

12- Fluctuant SNHL and vertigo usually occurs in:


a. Presbyacusis.
b. Meniere's disease.
c. Otosclerosis.
d. All of the above.

13-In a patient suffering from sudden severe vertigo lasting for 2 days
which resolves spontaneously, but hearing is normal, the diagnosis is likely
a. Meniere’s disease
b. Vestibular neuronitis
c. Benign paroxysmal positional verigo
d. Labyrinthitis .

14-In case of Meniere's disease with mild SNHL is treated by all the following
EXCEPT:
a. Medical treatment.
b. Labyrinthectomy.
c. Endolymphatic sac decompression.
d. Vestibular nerve section.

15- Destruction of right labyrinth causes nystagmus to


a. right side
b. Left side
c. rotatory nystagmus
d. No nystagmus

16- A 75-year-old woman was admitted following a fall. During an assessment


of her fall she complained of balance problems and dizziness whenever she
looked upwards. On examination, her gait and balance and neurological
examination was normal and there were no injuries. What is the most
likely diagnosis?
a. Benign paroxysmal positional vertigo
b. Cervical spondylosis
c. Concussion
d. Vertebrobasilar insufficiency.
17- A 75-year-old woman was admitted following a fall. During anassessment
of her fall she complained of balance problems and dizziness whenever
she looked upwards, for example, to hang laundry on her washing line outside.
On examination, her gait and balance and neurological examination was normal
and there were no injuries. What is therecommended management in this case?
a. Betahistine
b. Epley manoeuvre
c. Referral to neurology
d. Soft collar of the neck.

18- Meniere disease is a disorder characterized by recurring attacks of disabling


vertigo , fluctuating hearing loss in lower frequencies, and noise
in the ear (tinnitus). Which of the following is a possible cause of this disease?
a. A change in head position that moves calcium particles from one
part of the inner ear to another
b. A hereditary malformation of a portion of the inner ear
c. An excess of the fluid normally found in the inner ear
d. An infection of the inner ear.

19- A 35-year-old man has an 8-month history of fluctuating deafness,


fullnessand buzzing/ringing in both his ears. He also describes episodes
of vertigo, where the room is spinning around him and he feels extremely
nauseated. Rinne’s test demonstrates that air conduction is better than
bone conduction in both ears and Weber’stest does not show lateralisation.
What is the most likely diagnosis?
a. Benign paroxysmal positionalvertigo
b. Acoustic neuroma
c. Labyrinthitis
d. Otitis media
e. Meniere’s disease

20- Which of the following is not a typical feature of Ménière’s disease?


a. Sensorineural deafness.
b. Pulsatile tinnitus
c. Vertigo.
d. Fluctuating deafness

21- In BPPV
a. is provoked by certain head movements,
b. It is more common in males (2:1), and patients >40 years old
c. Most cases are secondary to head trauma
d. singular neurectomy is the first line of treatment
e. all are true

22- Which of the following is the most vestibulotoxic of all medications?


a. Vancomycin
b. Loop diuretics
c. Aminoglycosides
d. Antiepileptic

23- A 35-year-old woman has been complaining over the last 10 years of
attacks of incapacitating vertigo,tinnitus and decreased hearing. During the attack
there was a sense of aural fullness, the patient described it as if his ear is about
to explode. In between the attacks that usually occur once or twice a week the
patient feels fine or may have a minor sense of imbalance. The patient also
reported that her hearing ability has decreased over the years. Examination of
the ears showed bilateral normal tympanic membranes and some non-occluding earwax
A -What is the most likely diagnosis?
a. Benign Paroxysmal Positional vertigo
b. Endolymphatic hydrops
c. Acoustic neuroma
d. Vertebrobasilar insufficiency
e. Viral Labyrinthitis

B- Which of the following statements is MOST CORRECT regarding her diagnosis?


a. Patient will get relief if the Epley manouvre is performed
b. Vertigo never typically last more than just a few seconds
c. Increasing salt intake should improve her vertigo
d. A positive Dix-hall Pike Test confirms the disease
e. Low frequency hearing loss is usually seen on audiometry

24- All are features of Meniere’s disease except


a- Fluctuating hearing loss
b- Vertigo in certain head positions
c- Sense of fullness in the ear
d- Tinnitus.

25- Vertigo during ear wash is due to


a) Vagal stimulation
b) Very hot water
c) rupture of tympanic membrane
d) Injury of the external auditory canal

26-In a right serous labyrinthitis , nystagmus direction is


a) To the right side
b) To the left side
c) Has no specific direction
d) Vertical .

27- Which of the following is not a typical feature of Meniere’s disease


a- Fluctuating sensorineural hearing loss
b- vertigo
c- pulsatile tinnitus
d- tinnitus.

28- All of the following conditions produce nystagmus to right except


a- Labyrinthectomy on the left
b- Vestibular neuronitis on the right side
c- Serous labyrinthitis on the right side
d- Purulent labyrinthitis on the left

29- Dix Hallpike testing is diagnostic of


a- Benign paroxysmal positional vertigo
b- Meneiere’s disease
c- Vestibular neuronitis
d- Circumscribed labyrinthitis
Complicated ear

1) Intermittent fever with rigors and headache in a patient with


cholesteatma may be due to
a. otogenic meningitis
b. otogenic brain abscess
c. lateral sinus thrombophlebitis
d. extradural abscess.

2) Acute mastoiditis is manifested by all of the following EXCEPT:


a. Tenderness over mastoid antrum.
b. Continuous ear discharge.
c. Sagging of postero-superior meatal wall.
d. Obliteration of retro-auricular sulcus.

3) Fever, headache, vomiting & neck rigidity in a patient with cholesteatoma


is an indication of:
a. Lumbar puncture.
b. CT scan.
c. Fundus examination,
d. All of the above.

4) A positive Kernig's sign means


a. reflex flexion of the hips and knees when the neck is flexed
b. inability to extend the knee completely when the hip is flexed on the abdomen
c. inability to do rapid alternating movement
d. none of the above .

5) A positive Brudzniski sign means


a. reflex flexion of the hips and knees when the neck is flexed
b. inability to extend the knee completely when the hip is flexed on the abdomen
c. inability to do rapid alternating movement
d. none of the above.

6) A persistent profuse ear discharge after acute otitis media is


a. cholesteatoma
b. secretory otitis media.
c. Mastoiditis.
d. diffuse otitis externa.

7) The combination of unilateral otorrhoea, severe facial pain and diplopia is known as
a. Pierre Robin syndrome
b. Gradenigo’s syndrome
c. Kartagner's syndrome
d. Ramsay Hunt syndrome

8) A child with an attic drum perforation who developed nausea, projectile


vomiting and fever of 40 degree is suspicious to have got
a. otogenic meningitis
b. otogenic labyrinthitis
c. petrositis
d. mastoiditis.

9) In a case of cholesteatoma, severe spontaneous vertigo with nausea and


vomiting is suspicious of
a. circumscribed peri-labyrinthitis
b. diffuse serous labyrinthitis
c. extradural abscess
d. petrositis.

10) A 15 years old patient with right offensive otorrhoea, temperature of 35.5 C,
left body weakness, severe headache, vomiting, and visual field defects has:
a. Left Cerebellar abscess
b. Right cerebellar abscess
c. Right temporo-parietal abscess
d. Left temporoparietal abscess.

11) Diplopia on looking to the left side in a patient with a history of bilateral
chronic suppurative otitis media is likely due to
a. right petrositis
b. left lateral sinus thrombophlebitis.
c. right petrositis affecting the abducent nerve
d. left petrositis affecting the abducent nerve .

12) Griessinger’s sign:


a. is tenderness over the mastoid tip
b. is positive in Acute labyrinthitis .
c. is due to septic thrombosis of the cavernous sinus .
d. is erythema and edema over the posterior part of the mastoid due to
septic thrombosis of the lateral sinus.

13) The complication most feared in one year–old infant with acute suppuratives media is
a- Labyrinthitis
b- Mastoid abscess
c- Zygomatic abscess
d- Meningitis

14) Bezold’s abscess is a collection of pus


a- above and in front of the auricle.
b- behind the auricle.
c- in the upper part of the neck deep to the sternomastoid .
d- in the peritonsillar space

15) Cholesteatmoa causes fistula commonly in:


a. Promontory
b. Lateral semicircular cana
c. Posterior semicircular canal
d. Stapes footplate.

16)In Gradenigo's syndrome diplopia is due to inflammation of the following


cranial nerve
a. IV nerve
b. V nerve
c. III Nerve
d. VI nerve.

17) Extra cranial complications of chronic suppurative otitis media includes:


a. Mastoiditis..
b. Bezold's abscess..
c. Lateral sinus thrombosis.
d. Facial nerve paralysis

18) A patient with long standing left otorrhoea presented with persistent
left temporal headache, the diagnosis may be
a. migraine.
b. mastoiditis
c. extradural abscess.
d. meningitis.

19) Most cases of extradural abscess of the temporal lobe


a. Are asymptomatic
b. Present with persistent ipsilateral headache
c. Present with vertigo
d. Present wit pulsating ear discharge

20) Griessinger sign is positive in


a. Petrositis
b. Sinusitis
c. Labyrinthitis
d. Lateral sinus thrombophlebitis.
Facial nerve
1- The facial nerve nucleus is located at
a. Pons
b. Upper Medulla
c. Lower Medulla
d. Midbrain.

2- The facial nerve nucleus


a. is controlled by Pyramidal fibres from the cortical motor area.
b. The upper part of the nucleus receives pyramidal fibres from both sides,
c. the lower part of the nucleus receives pyramidal fibres from only the contralateral side.
d. Extra-pyramidal fibres for involuntary emotional movements as smiling & crying.
e. all

3- In lower motor neurone facial paralysis with normal Schirmer's test and
loss of taste sensation at the anterior 2/3 of the tongue, the level of the lesion is
a. in the internal auditory canal
b. in the horizontal tympanic part
c. at the Geniculate ganglion
d. in the stylomastoid foramen.

4- The facial nerve gets its seretomotor fibers from


a. superior salivatory nucleus
b. inferior salivatory nuscleus
c. nucleus solitaries
d. none

5- Taste sensation from the anterior two thirds of the tongue is carried by chorda
tympani facial nucleus to:
a. facial nucleus
b. superior salivatory nucleus
c. inferior salivatory nuscleus
d. nucleus solitaries

6- The following are causes of facial nerve paralysis


a- Vestibular schwannoma
b- Ramsay Hunt syndrome
c- Malignant otitis externa
d- All

7- A 70-year-old woman has weakness of the left side of her face. She has
had a painful ear for 48 hours. There are vesicles in the left ear canal and
on the ear drum.. Which of the following is associated with this condition?
a. Paracusis willisii
b. Furunculosis of the external auditory meatus
c. osteomyelitis of the temporal bone caused by pseudomonas
d. Paraganglioma arising from the middle ear
e. Reactivation of a Latent viral infection in the geniculate ganglion

8- In upper motor neurone facial paralysis


a. the upper part of the face is spared
b. the lower part of the face is spared
c. both upper half and lower half are paralyzed
d. hypotonia of facial muscles is characteristic of this type.

9- A patient has sustained a fracture of the skull base. Thorough examination


concludes that the greater superfacial petrosal nerve has been injured.
The conclusion was based on
a. partial dryness of the mouth due to lack of salivary secretion from the
submandibular and sublingual gland.
b. partial dryness of the mouth due to lack of salivary secretion from the parotid gland.
c. Dryness of the cornea due to lack of lacrimal secretion.
d. Loss of taste sensation from the anterior 2/3 of the tongue.

10-In lower motor neurone facial paralysis with intact taste sensation at
the anterior 2/3 of the tongue, the level of the lesion is
a. in the internal auditory canal
b. in the horizontal tympanic part
c. in the vertical part above the nerve to stapedius
d. in the stylomastoid foramen

11-In lesion of the facial nerve at the stylomastoid foramen, there is:
a. L.M.N.L of the facial muscles.
b. No impairment of taste.
c. No impairment of salivation.
d. No impairment of lacrimation.
e. All of the above.

12- Failure to close the eye voluntarily is a symptom of


a. paralysis of the trigeminal nerve
b. upper motor neurone facial paralysis
c. lower motor neurone facial paralysis
d. none of the above.

13-In lesion of the facial nerve at the stylomastoid foramen, there is:
a- L.M.N.L of the facial muscles.
b- No impairment of taste.
c- No impairment of salivation.
d- No impairment of lacrimation.
e- All of the above.

14-A patient with facial nerve paralysis suffers intolerance of loud sound due to
denervation of which muscle
a. posterior belly of digastrics
b. tensor tympani
c. stapedius muscle
d. tensor palatine muscle.

15-A preauricular swelling of 2 months duration and progressive course of facial paralysis is
a. Mumps
b. Malignant parotid tumor
c. Bell’s palsy
d. Temporo-mandibular joint arthritis.

16-Acute suppurative otitis media associated with facial paralysis is adequately treated by
a. Myringotomy and systemic antibiotic
b. Cortical mastoidectomy and systemic antibiotic
c. Cortical mastoidectomy with facial decompression
d. Systemic antibiotic.

17-In upper motor neurone facial paralysis


a. the upper part of the face is spared
b. the lower part of the face is spared
c. both upper half and lower half are paralyzed
d. hypotonia of facial muscles is characteristic of this type.
18-In upper motor neurone facial nerve paralysis
a- Paralysis occurs on the same side of the lesion.
b- Upper part of the face is not affected.
c- smiling is affected .
d- Muscles are flaccid.
e- No muscles fasciculation.

19-In lower motor facial neurone facial nerve pralysis


a- Both upper and lower parts of the face are affected .
b- involuntary movements are not affected.
c- Muscles are spastic .
d- Muscle fasciculation occur.

20-Lesion of the facial nerve at the geniculate ganglion is characterized by


a- No Loss of taste on the anterior 2/3 of the tongue .
b- No Loss of salivation from the submandibular salivary gland.
c- Phonophobia
d- No Loss of lacrimation..

21-A patient with facial nerve paralysis suffers intolerance of loud sound due to
denervation of which muscle
a. posterior belly of digastrics
b. tensor tympani
c. stapedius muscle
d. tensor palatine muscle.

22-In lower motor neurone facial paralysis with normal Schirmer's test and loss
of taste sensation at the anterior 2/3 of the tongue, the level of the lesion is
a. in the internal auditory canal
b. in the horizontal tympanic part
c. at the Geniculate ganglion
d. in the stylomastoid foramen.

23- A patient has sustained a fracture of the skull base. Thorough examination
concludes that the greater superfacial petrosal nerve has been injured.
The conclusion was based on
a. partial dryness of the mouth due to lack of salivary secretion from the
submandibular and sublingual gland.
b. partial dryness of the mouth due to lack of salivary secretion from the parotid gland.
c. Dryness of the cornea due to lack of lacrimal secretion.
d. Loss of taste sensation from the anterior 2/3 of the tongue.

24- The cause of Bell's palsy may be one of the following except:
a. Vascular ischemia.
b. Virus infection.
c. Bacterial infection.
d. Auto immune.

25- The early symptom of Bell's palsy is:


a. Dropping of angle of the affected side.
b. Obliteration of the angle of the mouth.
c. Pain of acute onset behind the ear.
d. Inability to close the eye.

26- The pathology in case of Bell's palsy is:


a. Facial nerve tumor.
b. Cut in the tympanic segment in the facial nerve.
c. Edema of the facial nerve inside its bony canal.
d. Hemorrhage in the facial nerve nucleus.

27-A 25-year-old man awoke to find that he is unable to close his left eye and is
unable to keep liquids from leaking out of his mouth. On inspection He
Has a flattened nasolabial fold and is unable to wrinkle the forehead on his
left side. His head and neck examination is unremarkable and he
has no other focal neurological deficits. Which of the following is
CORRECT, regarding his most likely diagnosis?
a. Diagnosis of exclusion
b. Treatment is with antibiotics
c. Facial nerve surgery is usually required
d. 90% of cases do not show spontaneous resolution
e. Anti-viral and Steroids medications are not recommended.

28- The most common cause of Bell’s palsy is:


a. Trauma to the temporal bone
b. Cholesteatoma
c. Glomus tympanicum
d. Idiopathic.

29- . In Bell’s palsy the edema of the facial nerve commonly affects the nerve at
a- Labyrinthine segment
b- Tympanic segment
c- Mastoid segment
d- At the stylomastoid foramen

30-A preauricular swelling of 2 months duration and progressive course of


facial paralysis is
a. Mumps
b. Malignant parotid tumor
c. Bell’s palsy
d. Temporo-mandibular joint arthritis .

31-In addition to facial paralysis, the most common finding in Ramsay-Hunt syndrome is
a. Otalgia
b. Ophthalmoplegia
c. Downbeat nystagmus
d. Conductive hearing loss
e. Pulsatile tinnitus.

32-In a patient having Right Deviation of the angle of the mouth and can
close his left eye properly with normal emotional face . The diagnosis is most probably
a- Right Upper motor neurone facial paralysis
b- Right Lower motor neurone facial paralysis
c- Left Upper motor neurone facial paralysis
d- Left upper motor neurone facial paralysis

33-In Ramsay Hunt syndrome, herpes zoster virus involves which ganglion
a- Geniculate ganglion
b- Sphenopalatine ganglion
c- Vestibular ganglion
d- None of the above

34-The most important in the management of Bell‘s palsy is


a- Care of the eye
b- Corticosteroids
c- Electroneurography
d- Decompression of the nerve.
MCQs

nose
ANATOMY AND PHYSIOLOGY

1- All about the vestibule of the nose all are true EXCEPT
a- It is the entrance to the nasal cavity.
b- It is lined with skin.
c- It contains sebaceous glands and hairs.
d- It is lined by mucus membrane of respiratory type.
e- Can be seen without nasal speculum.

2- Which of the following sinuses is related to the orbit


a- maxillary sinus
b- ethmoid sinuses
c- frontal sinus
d- sphenoid sinus
e- all of the above.

3- The anterior ethmoid sinuses drain into


a- the middle meatus.
b- the superior meatus.
c- the spheno-ethmoidal recess.
d- none

4- Mucosa of the nasal cavity is formed by all except:


a. Skin.
b. Columnar ciliated epithelium.
c. Olfactory epithelium.
b. Occasionally squamous epithelium.
d. Respiratory epithelium.

5- All of the following arteries share in Keisselbach’s plexus except:


a- anterior ethmoidal artery.
b- posterior ethmoidal artery.
c- greater palatine artery.
d- superior labial artery.
e- sphenopalatine artery.

6- The following Paranasal sinuses drain into the Ostiomeatal complex except
a. Posterior ethmoid sinuses.
b. Frontal sinus .
c. Maxillary sinus.
d. Anterior ethmoid sinuses.

7- The external carotid artery gives blood supply to the nose through
the following branches except
a- sphenopalatine artery
b- greater palatine artery
c- superior labial artery
d- anterior ethmoidal artery.

8- Anterior ethmoid presents bulge in the middle meatus called:


a- Bulla ethmoidalis.
b- Concha bullosa.
c- Lamina papyracea.
d- Haitaus semilunaris.

9- The key area in the middle meatus for drainage of anterior group of paranasal sinuses is
a- The ostiomeatal complex.
b- The osteomeatal complex.
c- The sphenoethmoidal recess.
d- The superior meatus.

10-The nasolacrimal duct opens into


a- superior meatus.
b- Middle meatus.
c- Inferior meatus.
d- Spheno-ethmoidal recess.

11-The teeth related to the floor of the maxillary sinus are


a- 1st premolar and 2nd premolar.
b- 2nd premolar and 1st molar.
c- 2nd and 3rd molar.
d- Canine and premolar.

12-The sphenoid sinus drains into


a- The middle meatus.
b- The inferior meatus.
c- The superior meatus.
d- The spheno-ethmoidal recess.

13-The parasympathetic nerve supply to the nose


a- Causes vasodilatation of the blood vessels.
b- Leads to increased nasal patency
c- Decrease the glandular secretion.
d- is through the deep petrosal nerve.

14- Olfactory cells are situated in


a- Middle third of the nasal cavity.
b- Upper third of the nasal cavity.
c- Lower third of the nasal cavity.
d- Anterior cranial fossa.

15- Before an odorant molecule can be perceived, it must first


a- contact the mucosa of the lower middle turbinate.
b- dissolve in the mucus overlying the olfactory receptors.
c- flow into the frontal sinus.
d- flow into the sphenoid sinus

16- Nasal respiration


a. neonates are obligatory nose breathing.
b. Important for warming the inspired air
c. The turbinates are dynamic tissue which change their size.
d. All are true .

17- The nasal septum is formed of the following EXCEPT


a. Vomer
b. Perpendicular plate of ethmoid
c. Quadrilateral cartilagre
d. Frontal process of maxillary bone

18- The frontal sinus


a. is absent at birth
b. its Growth ceases at about 20 years of age
c. is located on the frontal bone
d. all

19- Woodruff's plexus is a vascular network located on


a. The posterior lateral wall of the inferior meatus of the nasal cavity and it is generally
considered to be responsible for posterior epistaxis.
b. The anterior septum and responsible for anterior epistaxis
c. The anterior end of the inferior meatus and it is generally considered to be responsible for
anterior epistaxis.
d. None of the above.
e.

20- which of the following ganglia is related to lacrimation


a. otic
b. sphenopalatine
c. superior cervical
d. ciliary

21- All are true about sphenopalatine ganglion EXCEPT


a. it is located in the infratemporal fossa
b. it is anaesthetised by injection of local anaesthetic just above and
behind the posterior end of the middle turbinae
c. it is suspended from the maxillary nerve
d. it supplies secretomotor fibers to the nasal lacrimal and palatine glands
CHAPTER 1 EXAMINATION OF THE NOSE
CHAPTER 1 2022-2023

Examination of the nose

1- Some diseases affect special age groups


a- Nasopharyngeal angiofibroma affects children.
b- Antrochoanal polyp is mostly seen in old patients
c- Inverted papilloma is most commonly above forty
d- Squamous cell carcinoma affects commonly young adults.

2- The visible structures by anterior rhinoscopy are the following except


a- Anterior part of the nasal septum.
b- Anterior part of the lateral nasal wall.
c- Cribriform plate of the ethmoid.
d- Anterior part of the nasal floor.

3- The nasal the endoscopy has the following advantages


a. It is carried out in the out patient clinic under surface local anaesthesia.
b. It provides brilliant illumination
c. It allows examination of difficult-to-examine areas
d. It allows examination of areas which are inaccessible to clinical examination as
the meatii of the nasal cavities
e. all are true

4- Some diseases affects only males


a- Inverted papilloma
b- Nasopharyngeal angiofibroma.
c- Plummer Vinson syndrome
d- Postcricoid carcinoma.

5- The best investigation done for a patient suspected to have chronic sinusitis is
a- Coronal computerized tomography of the nose and sinuses
b- Occiptomental x ray
c- Magnetic resonance imaging.
d- Non of the above.

6- Biopsy is indicated in
a- Rhinoscleroma
b- Nasopharyngeal angiofibroma
c- Unilateral nasal polyp in an infant.
d- All

7- For nasopharyngeal examination


a- Anterior rhinoscopy is sufficient
b- Posterior mirror exaination is an easy and sufficient method.
c- Endoscopic examination is the best method
d. Radiologic evaluation is superior to the above methods.

Page 48
CHAPTER 1 EXAMINATION OF THE NOSE
CHAPTER 1 2022-2023

8- Regarding examination of the nose and sinuses


a. The superior turbinate is easily seen by turning up the tip of the nose with the thumb
b. Application of vasoconstrictor is not helpful in nasal examination
c. The use of flexible endoscopy is a good alternative
d. Middle meatus could be seen by displacing the middle turbinate medially .

9- The visible structures in anterior rhinoscopy are the following EXCEPT


a- The anterior part of Nasal septum
b- The anterior part of the Floor of the nose
c- The anterior part of Middle and inferior turbinate
d- The superior turbinate.

10- Posterior rhinoscopy by the traditional method using the postnasal mirror is not
preferred because
a. In some sensitive patients this prevents proper examinations.
b. Children are usually not very co-operative in this procedure.
c. gag reflex occurs in considerable percentage of patients
d. A nasopharyngoscopy (fibre-optic) is preferred
e. all

11- The best imaging modality to show the details of the nose and sinuses is
a. Plain X ray
b. CT scan
c. Ultrasonography
d. MRI

12- CT scan
a. A coronal CT image is the preferred initial procedure.
b. Bone window views provide excellent resolution and a good definition of the complete
osteomeatal complex and other anatomic details
c. The coronal view correlates best with findings from sinus surgery.
d. A noncontrast CT scan is usually sufficient, except for complicated acute sinusitis
e. all

13- MRI
a. can easily define bony anatomy as well as CT
b. only used to differentiate soft-tissue structures, such as in cases of suspected fungal
infection or neoplasm.
c. It uses ionizing radiation
d. All are true

Page 49
THE OBSTRUCTED NOSE

1- Foreign bodies in the nose


a- are most frequently found in the right nostril
b- are frequently found in children and the mentally retarded
c- The majority are diagnosed by inspection.
d- can usually be removed in the clinic except in uncooperative child
e- all

2- The enlarged inferior turbinate, all are true EXCEPT


a- may present as nasal obstruction
b- may result from a septal deviation in the wider side
c- can be treated with surgery
d- causes a bad smell in the nose

3- The following signs on CT scan are consistent with acute sinusitis EXCEPT :
a- Opaque maxillary antrum
b- Air/fluid level
c- Bone destruction
d- Mucosal thickening

4- Initially, bilateral choanal atresia is treated with


a- immediate emergency surgery
b- McGovern nipple.
c- Nasal catheters
d- Placing the infant on his or her abdomen.

5- Unilateral nasal obstruction in newly born infant may be due to:


a- Antrochoanal polyp.
b- Allergic nasal polyp.
c- Choanal atresia.
d- Non of the above.

6- Congenital choanal atresia is


a- Commonly unilateral
b- Commonly bony
c- A neonatal emergency, if bilateral
d- All are true.

7- congenital choanal atresia


a. most commonly membranous
b. most commonly bilateral
c. presents late if unilateral
d. all

8- An infant with bilateral choanal atresia presents with


a- Secretory otitis media
b- Respiratory distress.
c- Epistaxis
d- Stridor.

9- A bluish pulsatile, compressible mass which at the nasal dorsum of a 6 months


old child that increase with straining is most probably
a- Glioma
b- Encephalocele
c- Dermoid
d- Lipoma

10- Regarding a blocked nose/nasal obstruction in infancy, which of the following is false :
a. Foreign bodies may need removal using general anaesthesia
b. It is essential to test for catheter patency of nasal passages
c. Congenital nasal masses are not frequently found at birth
d. Antibiotics are the treatment of choice

11- A four years old child presented with left offensive nasal discharge. You should suspect
a- choanal atresia.
b- Adenoid.
c- foreign body impaction.
d- Rhinosinusitis.

12-In a patient presenting to the emergency room with fractured nose associated with edema
a- postpone reduction for one week .
b- immediate reduction of the nasal fracture is needed.
c- patient should be prepared for immediate septorhinoplasty .
d- postpone for one month before reduction

13- The triangular Fracture of the middle third of the face that runs through the
maxillary sinus, orbit, external nose and nasal septum is
a- Le Forte 1 fracture.
b- Le Forte 2 fracture.
c- Le Forte 3 fracture.
d- none of the above.

14- Fracture of the middle third of the face that runs transversely through the floor
of the maxillary sinus and nasal cavity is known as
a- Le Forte 1 fracture..
b- Le Forte 2 fracture.
c- Forte 3 fracture.
d- non of the above.

15- Septal hematoma, which is NOT TRUE


a. usually traumatic
b. may be due to blood dyscriasis
c. bilateral nasal obstruction is the commonest symptom
d. treatment is conservative

16-In septal abscess


a. pain is localized to the tip of the nose
b. systemic symptoms are unusual
c. treatment by antibiotic and analgesics is sufficient
d. a septal perforation may ensue

17- Patients with "aspirin intolerance" have a higher than normal risk of developing
a- nasal polyps
b- asthma
c- hyperplastic sinuses
d- negative skin test
e- all are true

18- The causative organism in influenza is


a- Haemophilus influenza
b- influenza virus type A, B, C
c- Rhinovirus
d- none of the above.

19- One of the following statements is NOT CORRECT concerning rhinosclerma


a- It is the commonest specific infective granuloma in Egypt.
b- The disease is endemic in Kalyobeyah.
c- Biopsy is needed to confirm diagnosis.
d- The causative organism is klebsiella ozynae.

20- On histopathological examination, the most diagnostic cells of rhinoscleroma are


a- monocytes
b- Russel’s bodies
c- Miculicz cells
d- Lymphocytes

21- Rhinoscleroma of the larynx mostly affects:


a- Posterior part of the larynx
b- Supraglottis
c- Anterior part of the larynx
d- Subglottis.

22- All the following lines of treatment could be applied in rhinoscleroma EXCEPT:
a- Rifampicin.
b- Cytotoxic drugs. .
c- Surgery to canalize the stenosed canal. .
d- Laser surgery.

23- Tuberculosis affects which part of the nasal septum


a- both cartilaginous and bony
b- cartilaginous
c- never affects the septum
d- bony portion

24-In a teenager presented with unilateral nasal obstruction with single pale
grayish glistening soft pedunculated mass. The most likely diagnosis is:
a- nasopharyngeal angiofibroma .
b- inverted papilloma .
c- antrochoanal polyp.
d- non of the above.

25- Syphilis of the nose may cause


a- bony septal perforation.
b- palatal perforation.
c- saddle nose deformity.
d- all of the above.

26- An atopic 35 years old patient presented with unilateral nasal obstruction.
Examination revealed unilateral multiple nasal polypi and CT scan
revealed unilateral sinus opacity with hyperdense spots . what is your diagnosis?
a- allergic nasal polypi
b- acute fulminating fungal sinusitis.
c- allergic fungal sinusitis
d- chronic indolent fungal sinusitis.

27- Juvenile nasopharyngeal angiofibroma is characterized by the following EXCEPT


a-Affects teenagers
b- Causes nasal obstructions
c-Very vascular tumor
d- Affects only females

28- The commonest benign tumor of the paranasal sinuses is


a. inverted papilloma
b. osteoma
c. nasopharyngeal angiofibroma .
d. non of the above.

29- All of the following are tumors of the


nose EXCEPT
a- Inverted papilloma.
b- Bleeding polyp of the septum.
c- Pott’s puffy tumor.
d- Osteoma.

30- Malignant tumors are common in which paranasal sinus


a- Maxillary sinus
b- Frontal sinus
c- Ethmoid sinus.
d- Sphenoid sinus.

31- Commonest site of inverted papilloma ofthe nose:


a- Lateral wall of the nose
b- Nasal septum c
c- Floor of the nose
d- Roof of the nose
the discharging nose

1- one of the following is a cause of bilateral nasal obstruction


a. foreign body
b. tumour
c. encephalocoele
d. large adenoids

2- Oroantral fistula may follow all of the following except


a. Extraction of 2nd premolar tooth.
b. Radical antrostomy operation..
c. Advanced maxillary carcinoma.
d. maxillary antrostomy .

3- Which of the following nasal foreign bodies is irritating and cause inflammatory reaction
a. bean
b. button.
c. Bead
d. a piece of plastic

4- Of the nasal foreign bodies one of the following may cause nasal septal perforation
a. metallic foreign body
b. vegetable foreign body
c. calculator battery.
d. Rhinolith

5- The most frequent cause of nasal allergy is


a. synthetic materials
b. smoke
c. insecticides
d. house dust mites

6- The organism involved in the pathogenesis of atrophic rhinitis is


a. klebsiella rhinoscleromatis .
b. klebsiella ozynae .
c. a and b .are true
d. neither a or b

7- In atrophic rhinitis the following signs are seen EXCEPT


a. reddish non ulcerating firm nodules at the muco-cutaneousjunction.
b. roomy nose.
c. greenish offensive crusts.
d. pale and atrophic mucosa

8- Cerebrospinal rhinorrhea is characterized by all of the following EXCEPT


a. watery and salty.
b. does not stiffen handkerchief.
c. increase on straining.
d. Bilateral.

9- The following are causes of unilateral offensive nasal discharge EXCEPT


a. nasal malignancy.
b. nasal foreign body.
c. nasal allergy.
d. Antrochoanal polyp

10- A 3 years old male child with recurrent epistaxis, subcutaneous hematoma and
swollen joints after minor trauma is probably suffering from
a. thrombocytopenic purpura .
b. hemophilia.
c. Leukemia.
d. rheumatic fever.

11- The mechanism of nasal allergy is:


a. Type 1 hypersensitivityreaction.
b. Type 2 hypersensitivityreaction.
c. Type 3 hypersensitivityreaction.
d. Type 4 hypersensitivityreaction.

12- Which is commonest cause of epistaxis in elderly is


a. Inflammation
b. Trauma
c. Sinusitis
d. Hypertension.

13- Sneezing is a prominent feature in:


a. Atrophic rhinitis.
b. Chronic rhinitis.
c. Allergic rhinitis.
d. Acute rhinitis

14- All the following is true about cerebrospinal fluid rhinorrhea EXCEPT:
a. clear fluid drips from the nose
b. initial treatment is to pack the nose
c. fluid contains glucose
d. fluid has a salty taste

15- All are true about the treatment of CSF rhinorrhea EXCEPT:
a. Antibiotics to avoid infection.
b. Nasal drops.
c. Treatment of the cause.
d. avoid straining .

16- Which of the following is used to confirm nasal allergy:


a. Esinophilia in nasal secretion.
b. Esinophilia in blood.
c. Elevated serum IGE.
d. All of the above.

17- Nasal regurgitation occurs in all of the following EXCEPT:


a. short soft palate
b. paralysed palate
c. redundant palate .
d. Cleft palate.

18- Unilateral mucopurulent & purulent nasal discharge may be due to:
a. Unilateral sinusitis.
b. FB in the nose.
c. unilateral choanal atresia .
d. All

19- The following drugs can cause epistaxis EXCEPT:


a. Salicylates.
b. Anticoagulants.
c. Quinine.
d. Ampicllin

20- The commonest cause of CSF rhinorrhea is:


a. Congenital.
b. Traumatic.
c. Infective.
d. Neoplastic.

21- Permanent Unilateral nasal discharge and unilateral nasal obstruction in 13 years
old boy is most probably diagnostic of :
a. Choanal atresia.
b. Adenoids.
c. Nasopharyngeal carcinoma.
d. Non of them.

22-In frontal sinusitis, the discharge is in:


a. The anterior part of the middle meatus.
b. The posterior part of the middle meatus.
c. All over the middle meatus.
d. The inferior meatus.

23-In maxillary sinusitis, the discharge is in:


a. The anterior part of the middle meatus.
b. The posterior part of the middle meatus.
c. All over the middle meatus.
d. The inferior meatus.
24- FB in the nose may be characterized by:
a. Unilateral nasal discharge.
b. Unilateral obstruction.
c. Unilateral epistaxis.
d. All of the above.

25- All are site of epistaxix EXCEPT


a. Little’s area
b. woodruff’s area
c. middle meatus
d. above the superior turbinate

26- The most common cause of epistaxis is


a. idiopatic
b. foreign body
c. tumor
d. trauma
e. vascular anomaly.

27- A 4-year-old boy presented with unilateral nasal bleeding and


chronic purulent drainge for 1 month . What is the most likely diagnosis ?
a. nasopharyngeal tumor
b. juvenile angiofibroma
c. septal deviation with maxillary sinusitis
d. foreign body
e. polyps.

28- A 17 year-old man had problem of frequent epistaxis. ENT exam reveal reddish
mass in Rt nasal cavity and nasopharynx. What is most likely diagnosis?
a. angiofibroma
b. nasal polyp.
c. inverted papilloma
d. nasopharyngeal carcinoma
e. rhinosinusitis.

29- A19-year-old young man had a problem of frequent epistaxis. ENT exam revealed
reddish mass in right nasal cavity and innasopharynx.Which of the followings does not
include in preoperative evaluation?
a. CT-scan
b. Endoscopic nasal &nasopharynx
c. angiography
d. paranasal sinus x-ray films
e. biopsy

30- The most likely source of bleeding in a 25-year-old man with severe
epistaxis 6 weeks after ethmoid, zygomatic fracture and ipsilateral blindness from
vehicle accident is the
a. anterior ethmoid artery
b. internal maxillary artery
c. sphenopalatine artery
d. internal carotid artery
e. posterior ethmoid artery

31- A 54-year-old man with hypertension has profuse epistaxis from posterior part
of Rt.nasal cavity which can not be controlled by anterior nasal packing.
The next step of management is :
a. arterial ligation
b. posterior nasal packing
c. anterior & post nasal packing
d. cauterization
e. arterial embolization.

32- lmmediate complication of anterior& posterior nasal packing include All, Except ?
a. Hypoxia
b. Eustachian tube dysfunction.
c. Hearing loss
d. Epiphora
e. Otitis media

33- All of the statements are true about posterior nasal packing except
a. It should be done in case of posterior epistaxis
b. Antibiotics should be prescribed to prevent infection in sinuses and middle ear
c. In aging patient O2 mark should be applied
d. Packing should be left in placed for about 3 days before removal
e. It should be done without anterior nasal packing

34- Some of the common complications of epitaxis include all of the following except
a. hypotension
b. aspiration
c. pulmonary embolism
d. hypoxia.
e. myocardial infarction

35- What is the most common cause of failure to control bleeding by nasal packing?
a. Use of a anterior and posterior balloon tampon.
b. Improperly placed packing
c. An occult bleeding diathesis.
d. A bleeding point inaccessible to the packing
e. Vaseline gauge is not appropriated for nasal packing

36- Clinical presentation of malignant nasopharyngeal disease may include all of


the following EXCEPT
a. lump in the neck
b. bloody nasal discharge
c. painful neck
d. deafness.

37- Extraction of which tooth commonly leads to oroantral fistula:


a. Second pre molar
b. First pre-molar
c. a and b
d. Second molar.
e. First molar

38- Sodium chromogycate is helpful in allergic rhinitis because it causes:


a. Desensitization
b. Decongestion of nasal mucosa
c. Most cell stabilization
d. Vasoconstriction.
the complicated sinuses

1- Uncontrolled diabetic patient was diagnosed as having acute sinusitis.


His general condition deteriorated rapidly. Nasal examination revealed blackish
crusts, the most likely diagnosis is
a. mycetoma.
b. acute fulminating fungal sinusitis.
c. allergic fungal sinusitis.
d. chronic indolent fungal sinusitis

2- All are true regarding mucocele of frontal sinus except?


a. Cystic tender swelling
b. Egg shell crackling can be elicited.
c. Displaces the eyeball downward and laterally.
d. Treatment is frontoethmoidectomy.

3- The following lesions may leads to proptosis EXCEPT:


a. Nasopharyngeal angiofibroma.
b. Nasopharyngeal carcinoma.
c. antrochoanal polyp
d. intraorbital bleeding in endoscopic sinus surgery

4- The commonest complication of sinusitis is:


a. Meningitis.
b. Orbital.
c. Extradural abscess.
d. Brain abscess.

5- Complications of sinusitis may include


a. mucocoele
b. blindness
c. death
d. deafness

6- Complications of sinusitis include the following EXCEPT


a. Pott’s Puffy tumor
b. frontal lobe abscess
c. Orbital cellulitis
d. cerebellar abscess
e. cavernous sinus thrombophlebitis

7- In preseptal orbital cellulitis


a. redness or discoloration of the eyelid,
b. Patients may be unable to open their eyes
c. visual acuity is not affected,
d. ocular movement is intact,
e. proptosis.

8- Orbital cellulitis is characterized by


a. redness and swelling of the eyelid
b. painful eye movement,
c. impaired vision .
d. limited eye movement. & proptsis
e. all

9- complications of following septal abscess include all EXCEPT


a. depression of the nasal bridge
b. meningitis
c. cavernous sinus thrombophlebitis
d. severe epistaxis

10- Ophthalmoplegia in Cavernous sinus thrombosis is due to paralysis of


a- occulomotor nerve
b- trochlear nerve
c- abducent nerve
d- all

11- In cavernous sinus thrombosis following sinusitis , the spread of infection is along
a. ophthalmic vein
b. angular vein
c. ptrygoid plexus
d. facial artery

12- Spread of infection in suppurative sinusitis


a. Osteitis occurs in compact bone.
b. Osteomyelitis occurs in diploic bone.
c. Meningitis usually arises via a septic thrombophlebitis.
d. all

13- Osteomyelitis of the frontal bone


a. Is most frequently due to frontal sinusitis.
b. Streptococci and staphylococci are the commonest bacteria responsible.
c. The onset is usually acute with rigors or epileptic fits.
d. Pott's puffy tumour is a forehead swelling due to frontal bone osteomyelitis with
associated subperiosteal abscess.

14- Orbital complications of suppurative sinusitis


a. All the paranasal sinuses border the orbit at some point.
b. Orbital complications are most common in children.
c. Orbital cellulitis usually precedes abscess formation.
d. Thrombophlebitis may result in cavernous sinus thrombosis.
e. all are true

15- Cavernous sinus thrombosis features include:


a. decrease or loss of vision,
b. proptosis
c. ophthalmoplegia
d. ptosis .
e. all

16- Ptosis in Cavernous sinus thrombosis is due to paralysis of


a. occulomotor nerve
b. trochlear nerve
c. abducent nerve
d. ophthalmic nerve

17- Dilated pupil in vavernous sinus thrombophlebitis is due to paralysis of


a- occulomotor nerve
b- trochlear nerve
c- abducent nerve
d- ophthalmic nerve
CHAPTER 5 SMELL DISORDERS
CHAPTER 1 2022-2023

smell disorders

1- Anosmia
a. must be bilateral before it is noticeable
b. it is often describes as a loss of taste
c. it may be due to brain tumor
d. all are true

2- Regarding disorders of smell, which is false


a. if caused by post influenza neuritis , the loss of smell is permanent
b. cacosmia may be due to foreign bosy in the nose
c. temporal lobe epilepsy may vause olfactory hallucination
d. ammonia is used to test the nerve .

3- In primary atrophic rhinitis


a. the patient suffers cacosmia
b. is commonest in old women
c. the nose is roomy and nasal obstruction seldom occur
d. Epistaxis is the commonest presenting symptom

4- Parosmia is
a. Inability to detect odors
b. Decreased ability to detect odors.
c. Altered perception of smell in the presence of an odor, usually unpleasant
d. smell of bad odor

5- in post infectious olfactory dysfunction, which is NOT TRUE?


a. it is typically caused by severe upper respiratory tract infection by a virus,
b. it is one of the most common causes.
c. The onset of perceived loss of smell is gradual
d. Patients often report on parosmia.

6- In Posttraumatic Olfactory Dysfunction


a. it is essential to rule out any brain, skull base, or nasal injuries
b. imaging is essential in managing the patient.
c. it is irreversible if no improvement occurs after 3 months
d. all

7- Anosmia
a. loss of smell or taste, are the predominant neurological symptom in Covid 19
b. Many countries list anosmia as an official COVID-19 symptom.
c. influenza neuritis is a common cause
d. all

Page 68
CHAPTER 6 RHINOGENIC & SINUGENIC FACIAL PAIN
CHAPTER 1 2022-2023

rhinogenic &sinogenic facial pain

1- in a patient presenting with severe throbbing pain in the nasal vestibule


a- the likely diagnosis is vestibulitis .
b- squeezing pus from the swelling is recommended.
c- the causative organism is streptococcus hemolyticus .
d- checking the blood sugar in recurrent cases is recommended.

2- A patient presented with bilateral nasal obstruction after nasal trauma. The patient
temperature is 38. There is throbbing nasal pain. Your diagnosis is
a- nasal furunculosis .
b- septal hematoma.
c- septal abscess.
d- non of the above.

3- A 25 years old patient with fever, mucopurulent nasal discharge and pain over the cheeks
is suffering from
a- Frontal sinusitis.
b- Acute maxillary sinusitis.
c- Ethmoidal sinusitis.
d- Chronic maxillary sinusitis

4- Headache in frontal sinusitis is due to:


a- Tension headache.
b- Vacuum headache.
c- Toxic headache.
d- All of the above.
5- The time of occurrence of headache in frontal sinusitis:
a- All the day.
b- At night.
c- Periodic.
d- Non of them.
6- The point of tenderness in acute frontal sinusitis is:
a- The inner canthus.
b- over the floor of the frontal sius
c. The infra-orbital margin.
d- Non of them.

Page 65
CHAPTER 6 RHINOGENIC & SINUGENIC FACIAL PAIN
CHAPTER 1 2022-2023

7- The point of tenderness in acute maxillary


sinusitis is:
a- The inner canthus.
b- The supra-orbital margin.
c- over the cheek
d- Non of them.

8- A young female presented with recurrent attacks of unilateral severe temporal


headache associated with nausea, vomiting and photophobia is likely to suffer
a. migraine
b. frontal sinusitis
c. tension headache
d. temporal arteritis

9- Unilateral, severe, sharp facial pain. precipitated by tooth brushing is likely


a. trigeminal neuralgia
b. frontal sinusitis
c. tension headache
d. temporal arteritis

10- Recurring attacks of severe sharp pain in the back of the throat and the area near the tonsils
which refers to the ear in otherwise normal oropharyngeal examination may be
a. infectious mononucleosis
b. acute tonsillitis
c. aphthous stomatitis
d. glossopharyngeal neuralgia.

Page 66
MCQs

pharynx
2022-2023 ANATOMY OF THE PHARYNX

anatomy of the pharynx

1- The part of the pharynx that lies in front of the 3rd to 6th cervical vertebra is
a- oropharynx
b- nasopharynx
c- hypopharynx
d- the whole pharynx.

2- The nasopharynx takes its sensory nerve supply from


a- trigeminal nerve
b- glossopharyngeal nerve
c- vagus nerve
d- non of the above

3- The posterior pillar of the tonsil is formed by


a- Palatoglossus muscle.
b- Palatopharyngeus muscle
c- Tensor palate muscle
d- Styloglossus muscle.

4- Infreior constrictor muscle of the pharynx take origin from


a- Hyoid bone
b- Mandible
c- Maxilla
d- Thyroid and cricoid cartilage

5- All the muscles of pharynx are supplied by pharyngeal branch of the vagus nerve
EXCEPT
a. stylopharyngeus
b. Palatopharyngeus
c. Salpingopharyngeus
d. Inferior constrictor

6- The soft palate muscles receive innervation by the pharyngeal plexus of the
vagus nerve, except
a. the tensor palati muscle
b. levator palati muscle
c. palatoglossus muscle
d. palatopharyngeus muscle

Page 70
2022-2023 ANATOMY OF THE PHARYNX

7- The oropharynx takes its sensory nerve supply from


a- trigeminal nerve
b- glossopharyngeal nerve
c- vagus nerve
d- non of the above

8- The hypopharynx takes its sensory nerve supply from


a- trigeminal nerve
b- glossopharyngeal nerve
c- vagus nerve
d- non of the above.

9- The voluntary stage of swallowing is


a- the oral phase
b- the pharyngeal phase
c- the esophageal phase
d- both 1 and 2

10- The second stage of swallowing is completed by


a- Elevation of the soft palate
b- Retroflexing epiglottis
c- Closure of the laryngeal aperture
d- Cricopharyngeal sphincter relaxation

11- How long is the pharynx?


a. 12cm
b. 20cm
c. 5cm
d. 18cm

12- The oropharynx


a- The roof is formed by the hard palate.
b- The posterior third of the tongue is a part of the oropharynx.
c- The oropharynx is lined by pesudostratified ciliated columnar epithelium.
d- The anterior pillar is made by the palatopharyngeal muscle.

13- The pharyngeal constrictor muscles, which is FALSE


a- The superior constrictor muscle forms a part of the tonsillar fossa.
b- Kilian dehiscenceis the gap between the middle and inferior constrictor muscle.
c- The motor nerve supply of constrictor muscles is from the pharyngeal plexus.
d- They are attached posteriory to the median raphe

Page 71
2022-2023 ANATOMY OF THE PHARYNX

14- Which of the following is a longitudinal muscle of pharynx ?


a. Superior constrictor
b. Thyropharyngeus
c. Cricopharyngeus
d. Stylopharyngeus

15- The palatine tonsils, which is NOT TRUE


a- Have crypts lined with squamous epithelium.
b- They have only efferent lymphatics.
c- Pain of tonsils may be referred to the ear through vagus nerve.
d- The tonsillar artery is a branch from the facial artery.

16- lymphatic drainage of the pharynx, which is NOT TRUE


a- The nasopharynx drains to the retropharyngeal lymph nodes.
b- The tonsils drain to the juguodigastric nodes.
c- The base of the tongus has very little lymphatic drainage.
d- all are true

17- What is wrong about the pharynx?


a. Extends from base of the skull to the sixth cervical vertebra at upper border of the
cricoid cartilage.
b. It is the upper part of the respiratory tract.
c. It is the upper part of the digestive tract.
d. About 10 cm in length in adult.
e. The pharyngeal cavity opens in front into the nose, mouth, and larynx.

18- All of the following statements are true about nasopharyngeal tonsil except:
a. They are lined by squamous epithelium.
b. They are present at birth and disappear by puberty.
c. They are situated in the roof and posterior wall of nasopharynx.
d. They do not have capsule.
e. They do not have crypts.

19- Pyriform fossa lies


a. Medial to aryepiglottic fold.
b. Lateral to aryepiglottic fold.
c. Inferior to aryepiglottic fold.
d. Superior to aryepiglottic fold.
e. None of the above.

Page 72
2022-2023 CHAPTER 1 PHARYNGEAL EXAMINATION

PHARYNGEAL EXAMINATION

1- Nasopharyngeal examination is best done by


a. digital palpation
b. mirror examination
c. flexible fiberoptic endoscopy
d. all

2- Regarding the tongue depressor


a. the metallic type is preferred
b. is used to depress the middle third of the tongue.
c. can be used to examine the hypopharynx by depressing the posterior third of the
tongue
d. all

3- Which of the following is used for oral examination


a. laryngeal mirror
b. rigid 70 degree endoscope is preferred
c. fiberoptic flexible endoscope
d. tongue depressor.

4- The direct laryngoscopic examination is used for all of the following EXCEPT
a. Examination of the hypopharynx and taking biopsy from any lesion.
b. Removal of FB
c. Removal of benign tumors.
d. Laryngeal examination.
e. excision of malignant tumors.

5- Barium swallow best demonstrates


a. the lumen of the pharynx
b. the surrounding structures
c. the surrounding lymph nodes
d. all

6- Absolute contraindications to mirror examination


a. Suspected epiglottitis
b. Ludwig angina
c. Angioedema
d. all

Page 74
2022-2023 CHAPTER 1 PHARYNGEAL EXAMINATION

7- The adverse effect of barium swallow is


a. Nausea and vomiting within 30 minutes of ingestion.
b. Hypersensitivity reactions
c. Extravasation of contrast into the mediastinum or from aspiration.
d. All

Page 75
2022-2023 CHAPTER 2 SORE THROAT

sore throat

1- A patient suffering from severe sore throat and generalized lymphadenopathy received
ampicilline injection by his family doctor and then he developed rubella like skin rashes.
You should consider doing
a- Blood picture
b- Abdominal ultrasound
c- Monospot test
d- All of the above are true
e- Continue giving ampicilline.

2- The most dangerous complication of ludwig`s angina is:


a. Acute laryngeal edema.
b. Dysphagia .
c. Bleeding.
d. Nasal obstruction.

3- The diagnostic manifestation of a Parapharyngeal abscess is


a- Trotter’s triad
b- Geisenger’s sign
c- Gradenigo triade
d- Boeck’s triade.

4- Boeck’s triad occurs due to:


a- Quinsy
b- Retropharyngeal abscess
c- Parapharyngeal abscess
d- Vincent angina.

5- Infection of the floor of the mouth which is usually dentogenic in origin


a- is called trench mouth
b- is known as Ludwig’s angina
c- is called Vincent’s angina
d- all are true

6- The site of incision for drainage of parapharyngeal abscess is :


a- Poaterior to the sternomastoid muscle.
b- Anterior to the sternomastoid muscles.
c- Through the crypta magna of the tonsil.
d- All of the above.

7- Acute retropharyngeal abscess is caused by:


a. TB of bodies of the cervical vertebrae.
b. Suppuration of the retropharyngeal gland.
c. Diphtheria bacilli.
d. Non of the above.

Page 76
2022-2023 CHAPTER 2 SORE THROAT

8- A 23-year-old student comes to Accident and emergency department with a complaint of


severe odynophagia, fever and left sided ear ache for a week. His voice is muffled and his
uvula is deviated to the right. The most likely diagnosis is
a. Infectious mononucleosis
b. Peritonsillar abscess
c. Peritonsillar carcinoma
d. Acute pharyngo-tonsillitis
e. Retropharyngeal abscess

9- A 21 year old woman presents with two day history of sore throat and
dysphagia. On examination she is pyrexial with halitosis and cervical
lymphadenopathy. What is the likely causative organism for her condition?
a. Epstein-Barr virus.
b. Agranulocytosis..
c. Corynebacterium diphtheriae.
d. Neisseria gonorrhoea.
e. Group A beta-haemolytic streptococcus (Streptococcus pyogenes)

10- Which of the following is NOT TRUE regarding acute reteropharyngeal abscess:
a. It is common in children.
b. Produces difficulty in breathing and suckling
c. It is limited to one side of the midline.
d. Incision and drainage is done from outside through carotid sheath.
e. It forms due to suppuration in reteropharyngeal lymph node.

11- A 30-year-old woman with a history diabetes mellitus and dental caries presents with a
painful swelling under the chin. She was prescribed antibiotics by her General
practitioner but did not take them consistently and subsequently developed difficulty
swallowing and trismus. Later she becomes febrile and hypoxic.
A- The most likely diagnosis is
a. Retropharyngeal abscess
b. parapharyngeal abscess
c. Ludwig’s angina
d. Vincent’s angina

B- The next best step is


a.N asotracheal intubation
b. Orotracheal intubation
c. Tracheostomy
d. Incision and drainage
e. Urgent CT scan of the Neck

Page 77
2022-2023 CHAPTER 2 SORE THROAT

12- The most common site of quinsy is:


a. Superior to tonsils.
b. Lateral to tonsils.
c. Posterior to tonsils.
d. Inferior to the tonsils.

13- . Trismus accompanying peritonsillar abscess is due to spasm of which muscle


a- Temporalis
b- Masseter
c- Medial pterygoid
d- Pharyngeal constrictor

14- An elderly diabetic patient developed milky white spots on his oral and pharyngeal
mucosa. You must suspect
a- Behcet disease
b- Aphthous ulcers
c- Vincent angina
d- Moniliasis.

15- Pharyngeal ulcer may be due to:


a. Syphilis.
b. Behcet's syndrome.
c. AIDS.
d. Allof the above.

16- Leucoplakia of the oral & pharyngeal mucosa is:


a. Malignant condition.
b. Precancerous condition.
c. Inflammatory condition.
d. Toxic condition.

17- The following antibiotic is contraindicated in infectious mononucleosis :


a. Ampicillin.
b. Erythromycin.
c. Cephalosporin.
d. Nonof the above.

18- Infection reaching the submental & submandibular space is called:


a. Vincentangina.
b. Ludwig'stumor.
c. Submandibular sialadenitis.
d. Bezold'sabscess.

Page 78
2022-2023 CHAPTER 2 SORE THROAT

19-Vincent angina is characterized by :


a. Very sever systemic symptoms & mild local symptoms.
b. Very sever local symptoms & mild systemic symptoms.
c. Very sever local & systemic symptoms.
d. Very mild local & systemic symptoms.

20- A 20- year-old man presents to the ENT clinic with sudden onset high fever of
one day duration, on examination: both tonsils are congested with whitish
membrane over the right tonsil with petichae over the palate and palpable both
jugulodiagastric lymph nodes. What is the most possible diagnosis?
a. Acute membranous tonsillitis.
b. Infectious mononucleosis.
c. Diphtheria.
d. Quinsy.
e. Leukaemia

21- Adenoid hypertrophy may lead to all of the following EXCEPT:


a. Adenoid face.
b. Otitis media with effusion.
c. Sensoryneural deafness.
d. Night mares.
e. Nocturnal enuresis

22- Acute tonsillitis may cause all of the following EXCEPT:


a. Acute retropharyngeal abscess.
b. Chronic retropharyngeal abscess.
c. Para Pharyngeal abscess.
d. Quinsy.

23- Chronic retropharyngeal abscess is treated by:


a. External drainage posterior to sternomastoid.
b. External drainage anterior to sternomastoid.
c. Internal drainage via longitudinal incision.
d. Non of the above.

24- Acute retropharyngeal abscess is treated by:


a. External drainage posterior to sternomastoid.
b. External drainage anterior to sternomastoid.
c. Internal drainage via longitudinal incision.
d. Non of the above.

Page 79
2022-2023 CHAPTER 2 SORE THROAT

25- Chronic retropharyngeal abscess is caused by:

a. TB of bodies of the cervical vertebrae.


b. Suppurationof the retropharyngeal gland.
c. Diphtheria bacilli.
d. Non of the above.

26- The most common cause for pharyngeal and oral ulceration is
a- Behcet disease.
b- aphthous ulcers.
c- tuberculous ulcers.
d- syphilitic ulcer.

27- Contraindication of tonsillectomy includes


a. Hemophilia.
b. Hypertrophic obstructing tonsils
c. Recurrent attacks of quinsy
d. Recurrent attacks of acute tonsillitis more than 6times/year fore 2 consecutive years

28- Behcet'sdisease is characterized by


a. Iridio-cyclitis.
b. Genital ulcers.
c. Progressive sensori-neural hearing loss.
d. Recurrent multiple small painful oral and pharyngeal ulcers which occur in groups.
e. All

29- Treatment of Behcet's disease consist of:


a. Corticosteroids locally & systemic.
b. Cytotoxic drugs.
c. Antihistaminic.
d. Non of the above

30- Attacks of stabbing pain in the tonsillar area suggest


a. Acute tonsillitis
b. Trigeminal neuralgia
c. Glossopharyngeal neuralgia
d. Quinsy.

31- Adenoid facies include all of the following except


a- Open mouth
b- Pinched nose
c- Crowding of teeth
d- Protruding tongue

Page 80
2022-2023 CHAPTER 2 SORE THROAT

32- Nasopharyngeal obstruction due to Adenoids can lead to all of the following except
a- Sinusitis
b- Otitis media with effusion
c- Adenoid facies
d- Rhinolalalia aperta

33- Waldeyer’s ring is situated in


a- Nasopharynx
b- Oropharynx
c- Tongue base
d- Both nasopharynx and oropharynx

34- Oral manifestations of HIV include


a- Oral candidiasis
b- Hairy leukoplakia
c- Recurrent aphthous ulcer
d- All

35- Plummer Vinson syndrome include all of the following EXCEPT


a- Koilonychias
b- Dysphagia
c- Atrophic glossitis
d- Oral moniliasis

36- Most common type of esophageal cancer is


a- Adenocarcinoma
b- Adenoid cystic carcinoma
c- Squamous cell carcinoma
d- Mucoepermoid carcinoma

Page 81
2022-2023 CHAPTER 3 DYSPHAGIA

dysphagia

1- Dysphagia means all of the following EXCEPT


a. difficulty with swallowing
b. a sensation of discomfort in the throat
c. discomfort initiated by pharyngeal movement
e. discomfort related to swallowing.

2- Globus pharyngeus is characterised by all of the following EXCEPT


a. difficulty in swallowing
b. a sensation of a lump in the throat
c. tightness of the chest
d. anxiety
e. hyperventilation

3- A middle aged female with gradually progressive dysphagia, koilonychia,


hypochromic anaemia and glazed tongue is suffering from
a- Plummer Vinson syndrome.
b- hypopharyngeal carcinoma.
c- oesophageal carcinoma.
d- Achalasia.

4- Moure’s sign can be detected in


a- Nasopharyngeal carcinoma
b- Oropharyngeal carcinoma
c- Postcricoid carcinoma
d- Supraglottic carcinoma

5- In a patient with progressive dysphagia to solids then also to fluids, you should
suspect all of the following except
a- Postcricoid carcinoma
b- Carcinoma of the pyriform fossa
c- Esophageal carcinoma
d- Achalasia of the cardia.

6- Which of the following tumours has the best prognosis


a- Postcricoid carcinoma
b- Nasopharyngeal carcinoma
c- Vocal fold carcinoma
d- esophageal carcinoma.

Page 83
2022-2023 CHAPTER 3 DYSPHAGIA

7- The Fossa of Rosenmullar is the common site for:


a- Angiofibroma
b- Lipoma
c- Adenoid
d- Nasopharyngeal carcinoma.

8- All of the following are precancerous lesions except


a- Plummer Vinson syndrome
b- Leukoplakia
c- adult solitary papilloma of the larynx
d- juvenile multiple papillomatosis of the larynx.

9- The pharyngeal pouch passes through:


a- Superior constrictor muscle
b- Killian dehiscence
c- Middle constrictor muscle
d- Hyoid bone.

10- A swelling felt in the neck which may gurgle on palpation, a sign known as
a- Boyce's sign.
b- Moure’s sign
c- Greisinger’s sign
d- kernig sign.

11- A 50 years old female has dysphagia for liquids and can swallow solids relatively well.
This suggests
a- Hypopharyngeal tumour.
b- Neurologic impairment.
c- Pharyngeal pouch.
d- Stenosis of the esophagus.

12- Esophageal carcinoma is investigated by


a. Barium swallow
b. CT scan
c. Endoscopy and biopsy
d. All of the above.

13- A 55-year-old woman presents with lethargy, easy fatigability and difficulty
swallowing. She has nail spooning, angular cheilitis and pale mucous membranes. A
barium swallow is requested and reveals a constriction at the crico-pharynx. The
most likely diagnosis is
a. Achalasia cardia
b. Dysphagia lusoria
c. Plummer Vinson syndrome
d. Oesophageal carcinoma

Page 84
2022-2023 CHAPTER 3 DYSPHAGIA

14- A 72y old man presents with intermittent difficulty in swallowing with regurgitation of
undigested food materials. Sometimes he wakes up at night with a feeling of suffocation.
Choose the single most likely cause of his dysphagia?
a. achalasia
b. plummer vinson syndrome
c. esophageal carcinoma
d. pharyngeal diverticulum

15- Plummer-Vinson syndrome is:


a. Malignant condition.
b. Pre-malignant.
c. Locally malignant.
d. Non of the above.

16- The dysphagia in plummer-vinson syndrome start to:


a. Solids then to fluids.
b. Fluids then to solids.
c. Fluid & solids at the same time.
d. all

17- The dysphagia in cardiac achalasia starts to:


a. Solids then to fluids.
b. Fluids then to solids.
c. Fluid & solids at the same time.
d. All of the above.

18- The anemia in Plummer Vinson syndrome is:


a. Microcytic hypochromic.
b. Macrocytic.
c. Both.
d. Non of them.

19- X-ray barium swallow showing tea-pot appearance is a finding in:


a. Plummer Vinson syndrome.
b. Cardiac achalasia.
c. Pharyngeal pouch.
d. Cancer oesophagus.

Page 85
2022-2023 CHAPTER 3 DYSPHAGIA

20- X-ray barium swallow showing rat-tail appearance is a finding in:


a. Corrosive oesophagitis.
b. Cardiac achalasia.
c. Pharyngeal pouch.
d. Cancer oesophagus.

21- X-ray barium swallow showing parrot-peak appearance is a finding in:


a. Plummer Vinson syndrome.
b. Cardiac achalasia.
c. Pharyngeal pouch.
d. Cancer oesophagus.

22- Dysphagia in cancer oesophagus is :


a. Progressive.
b. Regressive.
c. Stationary.
d. Intermittent.

23- Dysphagia in cardiac achalasia is:


a. Progressive.
b. Regressive.
c. Stationary.
d. Intermittent.

24- Dysphagia lusoria is:


a. Compression of the oesophagus by abnormally located RT subclavian artery
or double aorta.
b. Herniation of the pharyngeal mucosa via kllian dehiscence.
c. Chronic superficial oesophagitis with web formation.
d. Failure of relaxation of cardic sphincter.

25- X-ray with barium swallow showing multiple stricture in the oesophagus ia a
diagnostic finding in:
a. Chronic corrosive oesophagitis.
b. Cardiac achalasia.
c. Plummer Vinson syndrome.
d. Non of them.

26- Failure of relaxation of crico-pharyngeal sphincter during swallowing leads


to:
a. Plummer Vinson syndrome.
b. Cardiac achalasia.
c. Pharyngeal pouch.
d. Cancer oesophagus.

Page 86
2022-2023 CHAPTER 3 DYSPHAGIA

27- Dysphagia may be due to:


a. Aneurysm of the aorta.
b. Enlarged left atrium.
c. Mediastinal tumor.
d. All of the above.

28- Evaluation of swallowing disorders must include


a- history of symptoms
b- observation of swallow
c- barium swallow
d- oesophagoscopy
e- all

29- A three year old mongol boy get high fever then associated with difficult
swallowing and strider, the child was sitting , cannot sleep and drooling saliva,
the resident pediatrician sent for lateral cervical X-ray then he asked your
opinion about what was looking as a thumb in front of the hypopharynx, your
diagnosis was:
a. Acute pharyngitis.
b. Acute simple laryngitis.
c. Laryngo-tracheo-bronchitis.
d. Acute epiglottitis.
e. Croup.

30- What is not correct in acute epiglottitis?


a. Constant supervision in hospital is mandatory.
b. Dyspnea may be progressing and alarming.
c. It is a special form of acute laryngitis, in which the inflammatory changes affect
mainly the loosely attached mucosa of the epiglottis.
d. Systemic antibiotics is not a must to be started immediately.
e. Age incidence is between 1-6 year old.

Page 87
2022-2023 TONSILLECTOMY & ADENOIDECTOMY

tonsillectomy & adenoidectomy

1- In a 3 years old child with having mouth breathing, rhinolalia clausa and Snoring of one
year duration, the diagnosis is most likely,
a- bilateral choanal atresia
b- Adenoids
c- acute rhinitis
d- non of the above.

2- Tonsillectomy is urgently indicated in


a. children with moderate and severe obstructive sleep apnea
b. recurrent attacks of tonsillitis
c. quinsy
d. all

3- The first postoperative day, tonsillectomy fossa is:


a- Red colour.
b- Black colour.
c- Yellow colour.
d- White colour.
e- Flesh colour.

4- Trauma to posterior pillar during tonsillectomy causes:


a. Trismus.
b. Bleeding.
c. Infection.
d. Nasal regurgitation.
e. Ankyloglossia.

5- Tonsillectomy is indicated in all of the followingEXCEPT:


a. chronic hypertrophic tonsillitis
b. tonsillolith
c. acute tonsillitis
d. recurrent quinsy .

6- Pain in the ear in cases of acute tonsillitis or following tonsellictomy is referred via:
a- 5th nerve.
b- 9th nerve.
c- 10th nerve.
d- 12th nerve.

Page 89
2022-2023 TONSILLECTOMY & ADENOIDECTOMY

7- Hemorrhage within the 24 hours following tonsillectomy:


a- Primary.
b- Reactionary.
c- Secondary.
d- Non of the above.

8- Hemorrhage during the operation of tonsillectomy is:


a- Primary.
b- Reactionary.
c- Secondary.
d- Nonofthe above.

9- Hemorrhage 7days following tonsillectomy operation is:


a- Primary.
b- Reactionary.
c- Secondary.
d- Nonof the above.

10- The cause of reactionary hemorrhage after tonsillectomy:


a- Secondary infection.
b- Rising blood pr with slipping of ligature.
c- Injury of the pharyngeal muscles & mucosa.
d- Non of the above.

11- The cause of secondary hemorrhage after tonsillectomy :


a- Secondary infection.
b- Rising blood pr with slipping of ligature.
c- Injury of the pharyngeal muscles & mucosa.
d- Nonof the above.

12- The cause of suffocation &laryngeal spasm after tonsillectomy:


a- Backward fall of the tongue.
b- Inhalation of vomitus or blood clots.
c- Extubation spasm.
d- All of the above.

13- The value of post-tonsillectomy position is:


a- To prevent backward falling of tongue.
b- To prevent inhalation of the vomitus or blood.
c- To detect bleeding.
d- All of the above.

Page 90
2022-2023 TONSILLECTOMY & ADENOIDECTOMY

14- The most serious complication after tonsillectomy is


a. Hemorrhage
b. Respiratory obstruction
c. Shock
d. Infection.
15- In a 4 years old child presenting with bleeding per mouth 5 days after tonsillectomy
a. ligation of bleeding points should be carried out immediately.
b. the cause of bleeding in this patient is slippage of a loose ligature.
c. if conservative measures fail to stop bleeding consider packing the tonsillar bed with
absorbable haemostatic material
d. All of the above.

16- A 4 years old child presented by his mother complaining of lack of attention, mouth
breathing and hyponasality
a- secretory otitis media is suspected
b- adenoid is suspected
c- both a and b
d- non of the above

17- The following are signs of chronic tonsillitis except :


a- Enlarged cervical lymph nodes
b- Inequality of the size of the tonsils
c- Pus in the tonsillar crypts
d- Edema of the uvula.

18- A 9-year-old girl is scheduled for a day case tonsillectomy to alleviate symptoms of
upper airway obstruction and dysphagia. You are asked to examine the child and obtain
informed consent from her parents. Which of the following is a contraindication to
performing tonsillectomy in this patient?
a- Presence Cleft palate
b- Severe thrombocytopenia
c- Temperature of 39 degree C, with purulent rhinorrhea and inflamed tonsils
d- Wheezing and added breath sounds on auscultation.
e- all

Page 91
2022-2023 TONSILLECTOMY & ADENOIDECTOMY

19- A 5-year-old boy is brought to the clinic by his parents who complain that the child has
been snoring excessively for months. They are also concerned that he may have
decreased hearing and increased daytime sleepiness. The patient is scheduled for
tonsillectomy. However, on the day of surgery, he had a runny nose, temperature 38ºC
and dry cough. Which of the following should be the most appropriate decision for
surgery?
a. postpone surgery.
b. Proceed with surgery for 3 weeks and prescribe proper medicine
c. Proceed with surgery but give intra-operative steroids and antibiotics
d. Order a chest x-ray and ask pediatric medicine to review before proceeding with surgery.

20- An adenotonsillectomy was performed in a 5 years old child who was feverish and
having runny nose. Two hours after the procedure the recovery room nurse calls you
to review the patient because of excessive swallowing, hypotension and tachycardia. The
patient appears pale. What is the next best step in the management of this patient?
a- Apply an adrenaline soaked gauze to the bleeding area.
b- Commence intravenous fluids and observe evey 4-6 hourly.
c- Serial CBCs to monitor the hemoglobin levels.
d- Inject the tonsil bed with adrenaline solution.
e- Transfuse and examine patient under general anaesthesia.

21- An adenotonsillectomy was performed in a 5 years old child who was feverish and
having dry cough and runny nose. Six days post-surgery he is taken to the emergency
room because he spat up blood. This is most likely due to
a- Primary haemorrhage because of a dislodged clot in the tonsil bed.
b- Reactionary haemorrhaging due to weaning effects of the anaesthesia.
c- Secondary haemorrhaging associated with an upper respiratory tract infection
d- Haematemesis due to gastritis resulting from intraoperative steroids.
e- Slippage of ligatures used to tie the inferior pole

22- Which of the following is an indication for adenoidectomy


?
a. Velopharyngeal insufficiency
b. Recurrent epistaxis
c. Cleft palate.
d. snoring & Obstructive sleep apnoea

Page 92
MCQs

larynx
2022-2023 ANATOMY OF THE LARYNX

anatomy & physiology of the larynx

1. The respiratory tract is crossed by the isthmus of the thyroid gland at the level of the
a. cricoid cartlage
b. 1st and 2nd tracheal rings
c. 3rd and 4th tracharal rings
d. 5th and 6th tracheal rings .

2. The length of the larynx


a. about 4.5 cm in adult males and 3 cm in adult females
b. about 3.5 cm in adult males and 3.5 cm in adult females
c. about 6 cm in adult males and 5 cm in adult females
d. about 5 cm in adult males and 6 cm in adult females

3. Acute laryngitis in children is risky because


a. laryngeal lumen is smaller
b. laryngeal submucosa is looser
c. laryngeal cartilages are softer
d. laryngeal reflexes are immature
e. all

4. One of the following is not belonging to the laryngeal framework


a. thyroid cartilage
b. hyoid bone
c. cricoid cartilage
d. arytenoid cartilage
e. epiglottis .

5. The only complete ring of the respiratory tract is


a. thyroid cartilage
b. hyoid bone
c. cricoid cartilage
d. tracheal rings
e. epiglottis

Page 94
2022-2023 ANATOMY OF THE LARYNX

6. Adductor muscles of the larynx are all of the following EXCEPT


a- Lateral crico-arytenoid,
b- lateral Thyro-arytenoid,
c- Interarytenoid
d- posterior cricoarytenoid

7. Which muscle is tensor of the vocal folds


a. Lateral crico-arytenoid,
b. lateral Thyro-arytenoid,
c. Interarytenoid
d. posterior cricoarytenoid
e. cricothyroid

8. The only abductor muscle in the larynx is:


a- Sternothyroid muscle.
b- Lateral cricoarynoid muscle.
c- Cricothyroid muscle.
d Posterior cricoarynoid muscle.

9. The largest of laryngeal cartilages is


a- Thyroid cartilage
b- Cricoids cartilage
c- Epiglottis
d- Corniculate cartilage.

10. The cricothyroid muscle takes its nerve supply from


a- internal laryngeal branch of superior laryngeal nerve
b- external laryngeal branch of superior laryngeal nerve
c- internal laryngeal branch of recurrent laryngeal nerve
d- external laryngeal branch of recurrent laryngeal nerve

11. The mucosa of the upper larynx is innervated by the


a. internal laryngeal branch of superior laryngeal nerve
b. external laryngeal branch of superior laryngeal nerve
c. internal laryngeal branch of recurrent laryngeal nerve
d. external laryngeal branch of recurrent laryngeal nerve

Page 95
2022-2023 ANATOMY OF THE LARYNX

12. All are functions of the larynx except:


a. To protect the lungs.
b. To control air flow.
c. Phonation.
d. Generation of speech.
e. To build positive intrathoracic pressure.

13. Which of the following is the primary function of the larynx?


a. Phonation
b. Effort closure
c. Airway protection
d. Respiration
e. Valsalva.

14. Closure of the laryngeal sphincters during the second stage of swallowing occurs by
a. The ary-epiglottic
b. Ventricular folds
c. Vocal folds
d. All

15. The larynx prevents inhalation of fluids, food & foreign bodies into the lungs by
a. cessation of respiration during 2nd stage of swallowing
b. closure of laryngeal sphincter
c. elvation of the larynx backward tilt of the epiglottis
d. all.

16. Regarding the recurrent laryngeal nerve


a. it is a branch of glossopharyngeal; nerve
b. it rounds below the arch of aorta on the right side
c. it gives sensory innervation to all laryngeal mucosa
d. it supplies all muscles of the larynx
e. non of the above .

Page 96
2022-2023 CHAPTER 1 STRIDOR

stridor

1- Stridor:
a. It is an auditory manifestation of disordered respiratory function due to air
flow changes within the larynx, trachea, or bronchi.
b. It needs investigations in every case.
c. It is due to turbulence of air flow within a partially obstructed respiratory
tract.
d. It can be described in terms of its relationship to the phase of respiratory
cycle.
e. All the above.

2- Stridor at birth may be due to


a. laryngomalacia
b. large laryngeal web
c. large subglottic hemangioma
d. all

3- Laryngomalacia:
a. The larynx is of an exaggerated infantile t type.
b. The epiglottis is long and wide and folded backward at each lateral edge.
c. The epiglottis is converted into omega shaped incomplete cylinder.
d. The aryepiglottic folds are approximated .
e. all of the above.
.
4- Stridor at birth may be due to
a. laryngomalacia
b. small laryngeal web
c. bilateral choanal atresia
d. tracheoesophageal fistula.

5- What is NOT correct in acute eiglottitis


a. Constant supervision in hospital is mandatory.
b. Dyspnea may be progressing and alarming.
c. It is a special form of acute laryngitis, in which the inflammatory changes affect
mainly the loosely attached mucosa of the epiglottis.
d. Systemic antibiotics is not a must to be started immediately.
e. Age incidence is between 1-6 year old.

Page 98
2022-2023 CHAPTER 1 STRIDOR

6- An infant soon after birth develops inspiratory stridor which improves on lying on prone
position. The infant cry was normal. Temperature was 37, you should suspect
a- Laryngeal web.
b- Laryngomalacia.
c- Acute laryngitis.
d- Posterior laryngeal cleft.

7- All are causes of congenital strider EXCEPT


a. Acute epiglottitis.
b. Laryngeal web.
c. Subglottic stenosis.
d. Laryngomalacia.
e. Vascular anomaly.

8- The most common congenital laryngeal anomaly is


a- Laryngeal web.
b- Larygomalacia.
c- Subglottic stenosis.
d- Subglottic hemangioma.

9- A 3 years old boy complained of sudden acute respiratory distress, with spasmodic cough,
cyanosis & acting accessory respiratory muscles is most probably due to :
a. Acute follicular tonsillitis.
b. Foreign body inhalation.
c. Adenoid hypertrophy .
d. Vocal cord nodule .
e. Laryngeal web.

10- A 2 years old boy having acute laryngitis.


a- is treated at the outpatient clinic.
b- Hospitalization is essential
c- The condition is self limiting.
d- Antibiotic, corticosteroids and reassurance are sufficient

11- The causative organism of acute epiglottitis is


a- streptococcus pneumoniae
b- hemophylus influenza
c- staph aureus
d- Morexella cattarhalis.
2022-2023 CHAPTER 1 STRIDOR

12- Perichondritis of the larynx may be caused by


a- cut throat wound.
b- high tracheostomy .
c- radiotherapy.
d- all of the above.

13- laryngeal stenosis due to peri-chondritis may be due to


a- High tracheostomy.
b- Mid tracheostomy.
c- Low tracheostomy.
d- Prolonged use of a cuffed tube.

14- Scleroma of the larynx usually affects


a- The vocal cord
b- The subglottic region
c- The supraglottis
d- All

15- Respiratory distress may follow all of the following EXCEPT


a- Compression trauma of the larynx
b- Penetration trauma of the larynx
c- Inhalation of irritant gases
d- Abuse of voice.

16- The aim of Hemlich’s Manoeuvre is to


a- Move the larynx from side to side to assess for laryngeal click.
b- Apply a sudden sub-diaphragmatic upward thrust to produce artificial cough.
c- Forward pull of the mandible to clear the upper airway
d- None of the above is true.

17- A foreign body in the bronchus


a- is dislodged in the left bronchus more than the right bronchus.
b- Mostly seen in adults
c- Chest x ray and brochoscopic examination are recommended
d- None of the above.

18- The early Manifestations of laryngeal obstruction includes all of the following EXCEPT
a- Working ala nasi
b- Retraction of intercostals spaces.
c- Congested neck veins.
d- Cyanosis.

Page 100
2022-2023 CHAPTER 1 STRIDOR

19- Biphasic stridor is characteristic of


a. Laryngomalacia.
b. laryngeal web.
c. Acute laryngo-tracheobronchitis
d. epiglotitis
e. None

20- Inspiratory stridor is seen when the obstruction at


a- Below the level of the vocal cord
b- Above the level of the vocal cord
c- At the level of the vocal cord
d- at or above the level of the vocal cord.

21- About 1 mm edema reduces the normal laryngeal lumen by


a- 10%.
b- 20%.
c- 30%.
d 45%.

Page 101
2022-2023 CHAPTER 2 HOARSENESS OF VOICE

HOARSENESS OF VOICE

1- A new born baby presented with weak cry and hoarseness, the most probable
diagnosis:
a. Laryngomalacia.
b. Subglotic stenosis.
c. Subglotic heamangioma.
d. laryngeal web

2- A 50 years old male who presented with hoarseness of voice of more than one month
duration, should be subjected to
a- medical treatment and follow up.
b- endoscopic laryngeal examination.
c- vocal rehabilitation .
d- none of the above.

3- Aphonia and aspiration are symptoms of


a. unilateral abductor vocal cord paralysis .
b. bilateral abductor vocal cord paralysis.
c. bilateral adductor paralysis .
d. all of the above.

4- The earliest symptom to present in glottis carcinoma


a- Dyspnea.
b- Cough.
c- Hoarseness.
d- Aspiration

5- Hoarseness is an early symptom in


a. Glottis carcinoma
b. Supraglotic carcinoma
c. Subglottic carcinoma
d. Postcricoid carcinoma.

6- Laryngeal carcinoma commonly occurs in


a- elderly females
b- Elderly males
c- elderly smokers males
d- young adult males.

Page 103
2022-2023 CHAPTER 2 HOARSENESS OF VOICE

7- The commonest site of laryngeal carcinoma is


a. Supraglottis
b. Glottis
c. Subglottis
d. Laryngeal surface of the epiglottis.

8- Predisposing factors of laryngeal carcinoma include all of the following except


a. Smoking
b. Alcohol
c. Gastro-esophageal reflux
d. Hypochromic anemia.

9- A teacher presented with hoarseness of voice.On Laryngoscopic


examination,following image is seen.Which is the common location of this pathology
(marked by yellow arrow) ?

a. Anywhere
b. junction of anterior 2/3 and posterior 1/3 vocal cords
c. Junction of anterior 1/3 and posterior 2/3 vocal cords
d. Middle of anterior 1/3 and posterior 2/3 vocal cords

10- Dyspnea is an early symptom in


a. Glottis carcinoma
b. Supraglotic carcinoma
c. Subglottic carcinoma
d. Postcricoid carcinoma.

11- laryngeal condition is which O2 laser is almost mandatory is


a. Juvenille multiple paillomatosis
b. vocal nodule
c. cocal polyp
d. adult solitary papilloma

12- The usual treatment of laryngomalacia is


a. radiation
b. tracheostomy
c. steroids
d. reassurance

Page 104
2022-2023 CHAPTER 2 HOARSENESS OF VOICE

13- Lawyers and teachers are particularly likely to have vocal abuse and misuse
problems and may develop

a. singer nodule
b. Reink’s edema
c. leukoplakia
d. vocal cord polyp

14- Voice disorders is a feature of


a- glottis carcinoma
b- unilateral abductor paralysis
c- bilateral abductor paralysis
d- laryngoscleroma
e- all

15- A concerned mother brings her 3-week-old child to you because he is having
difficulties breathing. Upon laryngeal examination, a membrane has grown across
the anterior portion of the glottis. The diagnosis would be
a. hyperkeratosis
b. leukoplakia
c. papilloma
d. laryngeal web

16- A 40-year-old female teacher complaining of dysphonia for 4 months. Examination using
flexible fiber-optic scope showed a right vocal fold polyp. How are you going to treat this
patient?

a. Oral
b. Surgical excision
c. Voice rest
d. Voice therapy

17- A 5-year-old girl presented to pediatric hospital with biphasic stridor.. the most likely
diagnosis is ?
a. epiglottitis
b. laryngotracheobronchitis
c. subglottic hemangioma
d. scleroma

Page 105
2022-2023 CHAPTER 2 HOARSENESS OF VOICE

18- 30 years old lady underwent total thyroidectomy, after she woke up from the surgery and
started to talk she noticed changing in her voice. What is the most likely cause of her
hoarseness?
a. Infection
b. Thyrotoxicosis
c. Hypothyroidism
d. Vocal cord paralysis

19- A 5-year-old girl presented to pediatric hospital with biphasic stridor, temperature, 38
degreeC, No cyanosis but the child was irritable, the first step is
a. hospitalization
b. reassurance and medical treatment at home.
c. endotracheal intubation
d. tracheostomy

20- A 5-year-old girl presented to pediatric hospital with stridor, temperature, 39 degreeC, and
her voice was muffled, the first step is
a. examination of the pharynx to uncover thr diagnosis
b. indirect laryngoscopy
c. hospitalization and immediate start of IV antibiotic
d. reassurance and medical treatment at home.

Page 106
2022-2023 CHAPTER 3 RESTORATION OF THE AIRWAY

RESTORATION OF AN OBSTRUCTED AIRWAY

1- Head tilt/chin lift manoeuvre is used in


a. airway obstruction in conscious patient
b. airway obstruction in unconscious patient to keep the mouth open
c. airway obstruction in unconscious patient to prevent fall down of the tongue
d. suitable in cervical spine fracture

2- Which of the following protects the trachea from aspiration of secretions and stomach
contents?

a. Oropharyngeal airway
b. Nasopharyngeal airway
c. Cuffed endotracheal tube
d. All of the above.
e. None of the above.

3- The jaw thrust maneuver for opening the airway is an alternative of the head tilt-
chin lift maneuver in that:

a. It is easier to perform
b. It permits the use of either an oropharyngeal or nasopharyngeal airway
c. It can be performed without neck manipulation
d. It more closely approximates the natural airway anatomy

4- To perform a jaw thrust maneuver the physician:

a. Stands to the left of the patient, grasps the chin between the thumb and index
finger of the left hand and pulls the jaw forward
b. Stands at the patients head looking down at the patient, and pushes up on the
angle of the mandible using the middle fingers of each hand
c. Encourages the patient to thrust their jaw forward to maintain an open airway
d. Pushes down on the patients forehead with his left hand to extend the neck and
elevates the mandible using the tips of his right index and middle fingers.

Page 108
2022-2023 CHAPTER 3 RESTORATION OF THE AIRWAY

5- The nasopharyngeal airway has the advantage over the oropharyngeal airway in that
it:

a. Is made of soft plastic and therefore is less traumatic during insertion


b. Is usually better tolerated in a conscious or semi-conscious patient and can be inserted
in a patient who has their teeth clenched
c. Has a larger internal diameter and therefore allows a greater volume of gas exchange
d. Makes it easier to suction the oropharynx of excess secretions

6- Nasopharyngeal airways must be lubricated to ease insertion; you should use:


Select one:
a. any petroleum-based lubricant, such as WD-40.
b. petroleum jelly.
c. any silicone-based lubricant.
d. any water-based lubricant.

7- The nasopharyngeal airway is popular because it:

a. is made of rigid clear plastic, which is less likely to cause bleeding.


b. can be used even if clear (CSF) fluid is seen in the nose or ears.
c. often does not stimulate a gag reflex.
d. comes in more sizes than the oropharyngeal airway.

8- If you do not have the proper size oropharyngeal airway to fit your patient:
a. use the next larger size.
b. do not use one.
c. use the next smaller size.
d. use either a smaller or a larger one.

9- One method of determining which size oropharyngeal airway to use is by:


Select one:
a. use the largest airway that will fit into the patient's mouth.
b. make a visual comparison between the patient and the airway and pick the one that
seems closest.
c. comparing it to the diameter of the patient's little finger.
d. measuring the distance from the patient’s incisors and angle of the jaw

Page 109
2022-2023 CHAPTER 3 RESTORATION OF THE AIRWAY

10- How should you open the airway of an unconscious casualty?


a. Head tilt and chin lift.
b. Jaw thrust.
c. Head tilt and jaw thrust.
d. Lift the chin.

11- How long would you check to see if an unconscious casualty is breathing normally.
a. No more than 10 seconds.
b. Approximately 10 seconds.
c. Exactly 10 seconds.
d. At least 10 seconds.

12- Apnea immediately after opening the trachea is due to:


a. Rise of the blood carbon dioxide level.
b. Rise of the blood O2 level.
c. Wash of the blood carbon dioxide level.
d. Non of the above.

13- Indications of tracheostomy:


a. In all cases of acute laryngitis.
b. In any intubated patient within 3 days.
c. Unilateral choanal atresia.
d. Ludwig's angina .
e. If there is suspicion of laryngomalacia.

14- Care of tracheostomized patient include:


a. Humidification by wet gauze.
b. Changing the tube.
c. Care of the inflatable cuff.
d. All the above.

15- Post-operative care of tracheostomized patient:


a- constant attention is essential for the first 24 hours at least
b- Removal of secretions by suction.
c- Humidification: by wet gauze.
d- Care of the inflatable cuff.
e- all

Page 110
2022-2023 CHAPTER 3 RESTORATION OF THE AIRWAY

16- laryngeal stenosis due to perichondritis may be due to


a- High tracheostomy.
b- Mid tracheostomy.
c- Low tracheostomy.
d- Prolonged use of a cuffed tube.

17- The ideal site of tracheostomy is at


a- the1st and 2nd tracheal rings.
b- the 2nd and 3rd tracheal rings.
c- the 3rd and 4th tracheal ring.
d- the 4th and 5th tracheal rings.
e- the 5th and 5th tracheal rings.

18- The adult trachea


a- is about 10 cm length
b- is formed of 16-20 complete circular rings
c- has an inner diameter of about 1.5 to 2 centimetres.
d- All are true.
e- non is true

19- In a deeply comatose patient, tracheostomy is made to avoid


a. Bronchopneumonia
b. Aspiration pneumonia
c. Pulmonary edema
d. Respiratory failure.

20- In severe head injury, tracheostomy is performed


a- to prevent aspiration
b- for repeated suction.
c- For assisted ventilation
d- All

21- The aim of tracheostomy in obstructive sleep apnea is


a- to prevent aspiration
b- for repeated suction.
c- For assisted ventilation.
d- To by-pass the upper airway obstruction.

22- Which type of tracheostomy is preferred in cancer larynx


a- High tracheostomy.
b- Low tracheostomy.
c- Mid tracheostomy.
d- Any type is suitable .

Page 111
2022-2023 CHAPTER 3 RESTORATION OF THE AIRWAY

23- A patient complaining of persistent dyspnoea after tracheostomy, the tracheostomy


tube is clean, and there is diminished air entry on chest auscultation. You suspect
a- Surgical emphysema
b- Pneumothorax
c- Air embolism
d- All are true

24- About mid-tracheostomy one of the following statements is not correct


a- It is performed behind the thyroid isthmus
b- It is performed in the 3rd& 4thtracheal rings.
c- It is less liable to injure the dome of the pleura.
d- It is easy and rapid as compared to high tracheostomy.

Page 112

You might also like